You are on page 1of 97

www.insightsonindia.

com
www.insightsias.com
TEST 31 Solutions

Prelims 2016 Test 31


SOLUTIONS

1. The albedo of an object will determine its visual brightness when


viewed with reflected light. Consider the following statements
related to it.

Assertion (A): Mercury has higher Albedo than Earth.

Reason (R): Mercury is closer to the Sun as compared to earth.

In the context of the above, which of these is correct?

a) A is correct, and R is an appropriate explanation of A.


b) A is correct, but R is not an appropriate explanation of A.
c) A is correct, but R is incorrect.
d) A is incorrect, but R is correct.

Solution: d)
Justification: The planets are viewed by reflected sunlight and their
brightness depends upon the amount of light received from the sun and
their albedo.

Mercury receives the maximum amount of sunlight, but its albedo is only
0.1 so it is not as bright as it would be with a higher albedo.

Albedo of the Earth is 0.39 (Kaufmann) and this affects the equilibrium
temperature of the Earth. So, clearly A is incorrect. The greenhouse
effect, by trapping infrared radiation, can lower the albedo of the earth
and cause global warming.

Venus has the highest Albedo, of around 0.8, amongst all planets.

Q Source: Improvisation: Past year UPSC papers


www.insightsonindia.com
www.insightsias.com
TEST 31 Solutions

2. It prefers areas with agricultural lands, short bushes, scattered


trees in scrub forests and grassy plains. It hardly occurs in dense
forests. Major populations occur in the Terai lowlands in the
foothills of the Himalayas (northern India), Central and North-
western India. It has been declared as vermin in Bihar. The species
referred to here is?
a) Large Rock Rat
b) Giant squirrel
c) Nilgai
d) Wild Ass

Solution: c)
Justification: Large Rock Rat is a critically endangered species. Giant
squirrel is an endangered species. Both cant be declared vermins, as this
status allows the extermination of animals (after government
permission). So, (a) and (b) are clearly wrong.

The nilgai is indigenous to the Indian subcontinent and only one of


the four Indian antilopes that is still abundant. It is a least concer
category species as per IUCN.

Bihar was given permission by the Centre to kill Nilgais and wild
pigs as they destroyed standing crops.

For more than a decade, it has been legal for farmers in Madhya
Pradesh to kill a nilgai by seeking permission from district
authorities by citing continual damage to their standing crops.

Q Source: http://indianexpress.com/article/india/india-news-
india/to-save-crops-and-kill-nilgai-madhya-pradesh-renames-it-to-
rojad-in-rulebooks/
www.insightsonindia.com
www.insightsias.com
TEST 31 Solutions

3. Which of the following increases the credibility and strength of the


International monetary system?
1. Freely convertible global currencies
2. A mechanism to deal with Balance of payments (BoP) crisis

Which of the above is/are correct?

a) 1 only
b) 2 only
c) Both 1 and 2
d) None

Solution: c)
Justification: Statement 1: Foreign economic agents will accept a
national currency only if they are convinced that the currency will
maintain a stable purchasing power.

Without this confidence, a currency will not be used as an


international medium of exchange and unit of account since there
is no international authority with the power to force the use of a
particular currency in international transactions.

Governments have tried to gain confidence of potential users by


announcing that the national currency will be freely convertible at
a fixed price into another asset, over whose value the issuing
authority has no control. Hence, 1 is correct.

Statement 2: The international monetary system has been set up to


handle some of the above issues and ensure stability in international
transactions.

If there was no agent like IMF to settle BoP crisis of nations, global trade
and transaction would have received a setback in times of major crisis.

Even normally, the confidence levels in the global transactions would


have been low. Hence, 2 is correct.

Q Source: 12th NCERT: Macroeconomics


www.insightsonindia.com
www.insightsias.com
TEST 31 Solutions

4. NAMA-11 (Nama-11) group of countries is often mentioned in the


news in context of?
a) Missile Technology Control Regime
b) International Energy Agency
c) World Economic Forum
d) World Trade Organization

Solution: d)
Learning: It is a group of eleven developing countries working toward
strengthening Non-agricultural market access (NAMA) in WTO.

The group has two main objectives:

supporting flexibilities for developing countries


balance between NAMA and other areas under negotiation.

Member countries of NAMA-11 are Argentina, Bolivarian Republic of


Venezuela, Brazil, Egypt, India, Indonesia, Namibia, Philippines, South
Africa and Tunisia.

The Non-Agricultural Market Access (NAMA) negotiations of the World


Trade Organization are based on the Doha Declaration of 2001 that calls
for a reduction or elimination in tariffs, particularly on exportable goods
of interest to developing countries.

The WTO considers the NAMA negotiations important because NAMA


products account for almost 90% of the world's merchandise exports.

Q Source: In news

5. With reference to the order of evolution of living organisms, which


one of the following is correct?
1. Frogs evolved before Dolphins and whales.
2. Salmander evolved after Kangaroos.
3. Bats evolved before pigeons.

Select the correct answer using the codes below.


www.insightsonindia.com
www.insightsias.com
TEST 31 Solutions

a) 1 and 2 only
b) 1 only
c) 2 and 3 only
d) 1 and 3 only

Solution: a)
Justification: This is how broadly life evolved on earth.

About 500 million years ago, plants and fungi colonised the land
and were soon followed by arthropods and other animals.
Amphibians first appeared around 364 million years ago, followed
by early amniotes and birds around 155 million years ago (both
from "reptile"-like lineages), mammals around 129 million years
ago, homininae around 10 million years ago and modern humans
around 250,000 years ago.

So, one thing is very clear is that the order of evolution (increasing) is:

Plants and fungi < Arthopods and other animals < Amphibians
(frog, salamander) < Amniotes (crocodile) and birds (pigeon)
< Mammals (e.g. whales, bat, dolphin) < Humans

Based on this order, the correct answer is A.

Learning: However, despite the evolution of these large animals,


smaller organisms similar to the types that evolved early in this process
continue to be highly successful and dominate the Earth, with the
majority of both biomass and species being prokaryotes.

Q Source: Evolution-based questions asked in IES

6. The Government of India Act of 1935 provided for the


establishment of an All India Federation to be based on a Union of
the provinces of British India and the Princely States. But, the
federation never came into being because
a) Required number of princely states did not join it.
b) The GoI Act 1935 was later annulled.
c) Gandhiji and other national leaders were opposed to it.
d) Second World War broke out soon after.
www.insightsonindia.com
www.insightsias.com
TEST 31 Solutions

Solution: a)
Justification: The proposed all India federation included 11 provinces
of British India, 6 Chief Commissioners Provinces and those princely
states who might accede to the federation.

The term on which a state joined the Federation were to be laid


down in the Instrument of Accession.
Joining the federation was compulsory for the British Provinces
and chief commissioners provinces.
For princely states, the accession to the Federation was voluntary.
The federation could not be established until:
o A number of states, the rulers whereof were entitled to
choose not less than half of the 104 seats of the council of
state, and the aggregate population whereof amounted to be
at least one half of the total population of all the Indian stales
had acceded to the federation.

Princely states never joined, and thus the federation never came into
being. So, (a) is correct.

Learning: The Government of India Act 1935 derived material from


four key sources viz. Report of the Simon Commission, discussions at the
Third Round Table Conference, the White Paper of 1933 and the reports
of the Joint select committees.

This act ended the system of dyarchy introduced by GOI Act 1919 in
provinces and introduced it in the centre.

Q Source: Based on UPSC papers

7. Consider the following about the history of rocket technology in


India.
www.insightsonindia.com
www.insightsias.com
TEST 31 Solutions

Assertion (A): Rockets were used in warfare in India even before they
were used in Europe.

Reason (R): Europe supplied saltpetre to India especially to Gujarat


since the 15th Century.

In the context of the above, which of these is correct?

a) A is correct, and R is an appropriate explanation of A.


b) A is correct, but R is not an appropriate explanation of A.
c) A is correct, but R is incorrect.
d) Both A and R are incorrect.

Solution: c)
Justification: In A History of Greek Fire and Gunpowder, James
Riddick Partington describes the gunpowder warfare of 16th and 17th
century Mughal India, and writes that "Indian war rockets were
formidable weapons before such rockets were used in Europe. They had
bamboo rods, a rocket-body lashed to the rod, and iron points. So,
clearly A is correct.

The first iron-cased rockets were developed in the late 18th century
in the Kingdom of Mysore, adopted and improved as the Congreve
rocket and used in the Napoleonic Wars.

Gujart supplied Europe saltpeter for use in gunpowder warfare


during the 17th century. Bengal and Mlwa participated in
saltpeter production. The Dutch, French, Portuguese, and English
used Chhapra as a center of saltpeter refining. So, clearly R is
incorrect.

Learning: The following traces the history of gunpowder and war


rocket technology in India.

Gunpowder and gunpowder weapons were transmitted to India through


the Mongol invasions of India.

The Mongols were defeated by Alauddin Khilji of the Delhi Sultanate,


and some of the Mongol soldiers remained in northern India after their
conversion to Islam.
www.insightsonindia.com
www.insightsias.com
TEST 31 Solutions

It was written in the Tarikh-i Firishta (16061607) that the envoy


of the Mongol ruler Hulagu Khan was presented with a
pyrotechnics display upon his arrival in Delhi in 1258 CE.

As a part of an embassy to India by Timurid leader Shah Rukh


(14051447), 'Abd al-Razzaq mentioned naphtha-throwers
mounted on elephants and a variety of pyrotechnics put on display.

Firearms known as top-o-tufak also existed in the Vijayanagara


Empire by as early as 1366 CE.

From then on the employment of gunpowder warfare in the region


was prevalent, with events such as the siege of Belgaum in 1473 CE
by the Sultan Muhammad Shah Bahmani.

The use of mines and counter-mines with explosive charges of


gunpowder is mentioned for the times of Akbar and Jahngir.

By the 16th century, Indians were manufacturing a diverse variety


of firearms; large guns in particular, became visible in Tanjore,
Dacca, Bijapur and Murshidabad.

Q Source: Improvisation: Past year UPSC papers

8. Consider the following about Anekantavada.


1. It is a core theory and philosophy of Buddhism.
2. It says that the World is immaterial, infinite but mortal.

Which of the above is/are correct?

a) 1 only
b) 2 only
c) Both 1 and 2
d) None

Solution: d)
www.insightsonindia.com
www.insightsias.com
TEST 31 Solutions

Justification: It refers to the principles of pluralism and multiplicity of


viewpoints, or vantage points, the notion that reality is perceived
differently from diverse points of view, and that no single point of view is
the complete truth, yet taken together they comprise the complete truth.

It is one of the most important and fundamental doctrines of Jainism.

Jains contrast all attempts to proclaim the sole monopoly on truth with
andhagajanyyah, which can be illustrated through the parable of the
"blind men and an elephant".

Q Source: Past year papers UPSC

9. The often used term fourth estate of a democracy refers to the


a) Inter-governmental organizations
b) Political parties
c) Local Bodies
d) Press and Media

Solution: d)
Learning: Journalism has long been regarded as an important force in
government, so vital to the functioning of a democracy that it has been
portrayed as an integral component of democracy itself. In 1841, Thomas
Carlyle wrote, Burke said there were Three Estates in Parliament; but,
in the Reporters Gallery yonder, there sat a Fourth Estate more
important far than they all.

Democracy requires informed citizens. No governing body can be


expected to operate well without knowledge of the issues on which
it is to rule, and rule by the people entails that the people should be
informed.
www.insightsonindia.com
www.insightsias.com
TEST 31 Solutions

In a representative democracy, the role of the press is twofold: it


both informs citizens and sets up a feedback loop between the
government and voters.

The press makes the actions of the government known to the


public, and voters who disapprove of current trends in policy can
take corrective action in the next election.

Without the press, the feedback loop is broken and the government
is no longer accountable to the people. The press is therefore of the
utmost importance in a representative democracy.

Q Source: General questions on democracy

10. Ethylene, as a plant growth regulator, primarily plays which


of the following roles?
a) Breaking dormancy and inducing ripening in fruits
b) Protecting seeds from insects and pests
c) Increasing the hardness of the fruit after separation from the
tree
d) Transporting air and other vital gases from atmosphere to
the plant cells

Solution: a)
Justification: Ethylene is formed naturally in plants in amounts
sufficient to bring about regulatory effects. Role of ethylene is in:

Breaking dormancy

Induce ripening of fruits

Induce abscission of leaves

Inhibit elongation and lateral bud growth


www.insightsonindia.com
www.insightsias.com
TEST 31 Solutions

Learning: Ripening is the process by which fruits attain their desirable


flavour, quality, colour, palatable nature and other textural properties.

Ripening is associated with change in composition i.e. conversion of


starch to sugar.

On the basis of ripening behavior, fruits are classified as climacteric and


non-climacteric fruits.

Climacteric: Climacteric fruits are defined as fruits that enter


climacteric phase after harvest i.e. they continue to ripen. During
the ripening process the fruits emit ethylene along with increased
rate of respiration.

Ripe fruits are soft and delicate and generally cannot withstand
rigours of transport and repeated handling. These fruits are
harvested hard and green, but fully mature and are ripened near
consumption areas. Small dose of ethylene is used to induce
ripening process under controlled conditions of temperature and
humidity.

Q Source: Agritech website

11. Consider the following about Home Rule League movement in


India.
1. Two Home Rule Leagues were established, one by Tilak at
Poona and the other by Annie Besant at Madras.
2. The league was highly antagonistic towards Indian National
Congress (INC) and its policies.
3. The August Declaration by Montague in 1917 led to the end of
the Home Rule Movement.

Select the correct answer using the codes below.

a) 1 and 2 only
www.insightsonindia.com
www.insightsias.com
TEST 31 Solutions

b) 2 only
c) 1 and 3 only
d) 2 and 3 only

Solution: c)
Justification: Statement 1: While Tilaks Movement concentrated on
Maharashtra, Annie Besants Movement covered the rest of the country
starting with Madras.

The aim of the Movement was to get self-government for India within
the British Empire. It believed freedom was the natural right of all
nations.

Statement 2: The leaders of the Home Movement thought that Indias


resources were not being used for her needs.

The two Leagues cooperated with each other as well with the Congress
and the Muslim League in putting their demand for home rule.

Statement 3: The Home Rule Movement had brought a new life in the
national movement. There was a revival of Swadeshi. Women joined in
larger numbers.

In August 1917, Montague, the Secretary of State in England, made


a declaration in the Parliament of England on British
Governments policy towards future political reforms in India.

He promised the gradual development of self-governing


institutions in India. This August Declaration led to the end of the
Home Rule Movement.

Q Source: 12th TamilNadu History Textbook

12.Which of the following statements is INCORRECT about Lok


Adalats?
a) They are informal courts setup by the panchayat community.
www.insightsonindia.com
www.insightsias.com
TEST 31 Solutions

b) Cases at pre-litigation stage or pending in a court of law can


be settled by the Lok Adalat.
c) The award made by the Lok Adalats is deemed to be the
decree of a civil court.
d) All of the above

Solution: a)
Justification & Learning: It is a forum where the disputes/cases
pending in the court of law or at pre-litigation stage are
settled/compromised amicably. So, (b) is correct.

The Lok Adalat has been given statutory status under the Legal Services
Authorities Act, 1987. So, (a) is incorrect.

Under the said Act, the award made by the Lok Adalats is deemed to be
the decree of a civil court and is final and binding on all parties and no
appeal lies before any court against its award.

Nature of cases to be referred to lok adalat

Any case pending before any court


Any dispute which has not been brought before any court and is
likely to be filed before the court

Q Source: NALSA Website

13. The first telegraph line in India was laid between


a) Calcutta and Diamond Harbour
b) Bombay and Kandla
c) Ivory Coast and Lonawala
d) Gandhinagar and Ahmadabad

Solution: a)
www.insightsonindia.com
www.insightsias.com
TEST 31 Solutions

Learning: Lord Dalhousie paved the way for the Imperial Telegraph
Department in 1850.

A year later, British India's first telegraph line and office was opened in
October 1851, between Calcutta and Diamond Harbour along the busy
shipping route on the Hooghly.

By March 1854, there were 800 miles of telegraph lines between Calcutta
and Agra and this was further connected to Bombay and Madras.

Overseas telegraph communication was made possible in 1865 by


running cables along the seabed.

Q Source: Closure of Telegraph services by the government few years


back + Past year UPSC papers

14.Consider the following statements.


1. The largest inland saline wetland is Chilka Lake located in
Odisha.
2. The largest lagoon of India is Vembanad lake located in Kerala.

Which of the above is/are correct?

a) 1 only
b) 2 only
c) Both 1 and 2
d) None

Solution: d)
Justification: Statement 1: The largest inland saline wetland is
Sambhar Lake located in Rajasthan.

It is geographically a separate land locked river basin and a major source


of Rajasthan's salt production.

Statement 2: Chilka is a freshwater lagoon, and Indias largest.


Vembanad lake is Keralas largest lake.
www.insightsonindia.com
www.insightsias.com
TEST 31 Solutions

The Nehru Trophy Boat Race is conducted in a portion of the Vembanad


lake. It is also a Ramsar site and a popular tourist destination.

Q Source: Important wetlands in India

15. Consider the following statements.

Assertion (A): India has no privately managed seaports.

Reason (R): The Indian government does not allow private investment in
seaports for strategic reasons.

In the context of the above, which of these is correct?

a) A is correct, and R is an appropriate explanation of A.


b) A is correct, but R is not an appropriate explanation of A.
c) A is correct, but R is incorrect.
d) Both A and R are incorrect.

Solution: d)
Justification: Thirteen ports have been notified as major ports, all of
which are government owned. Ennore port is the only corporatised
major port in which the Centre holds a stake of about 68 per cent,
remaining 32 per cent is held by the Chennai Port Trust..

Ennore Port, officially renamed Kamarajar Port Limited, is located on


the Coromandel Coast and the first port in India which is a public
company.

The port has been able to attract an investment of 26,000 million by


private entrepreneurs on various terminals and harbour craft.

Q Source: Improvisation: CAPF 2016


www.insightsonindia.com
www.insightsias.com
TEST 31 Solutions

16.Consider the following about the Large Underground Xenon dark-


matter experiment (LUX).
1. It aims to capture dark matter.
2. It is situated a mile deep within earth.
3. It is operated at night to reduce the background noise of cosmic
rays.

Select the correct answer using the codes below.

a) 1 and 3 only
b) 3 only
c) 1 only
d) 2 and 3 only

Solution: b)
Justification: Statement 1: It aims to directly detect (not capture)
weakly interacting massive particle (WIMP) dark matter interactions
with ordinary matter on Earth.

Despite the wealth of (gravitational) evidence supporting the existence of


non-baryonic dark matter in the Universe, dark matter particles in our
galaxy have never been directly detected in an experiment.

Statement 2: It rests a mile deep (1.6 km) underground in a former


South Dakota gold mine that is now called the Sanford Underground
Research Facility.

Statement 3: Suspended in a huge tank of purified water, a titanium


tank holds frigid liquid xenon.

The xenon's job is to light up, with a jolt of electrical charge and a
faint flash of light caught by surrounding sensors, when a dark-
matter particle collides with one of its atoms and the gallons of
water and mile of rock's job is to stop anything else from getting in
and disturbing it.

It is operated underground to reduce the background noise signal


caused by high-energy cosmic rays at the Earth's surface.
www.insightsonindia.com
www.insightsias.com
TEST 31 Solutions

Q Source: http://www.space.com/33497-dark-matter-search-comes-
up-empty-lux-detector.html

17. Consider the following with reference to Ancient India.


1. The ancient town of Takshasila was located between Indus and
Jhelum.
2. The Pala rulers of Eastern India were patrons of Buddhism.
3. Kushanas ruled India after Greeks.

Select the correct answer using the codes below.

a) 1 and 2 only
b) 2 and 3 only
c) 1 only
d) 1, 2 and 3

Solution: d)
www.insightsonindia.com
www.insightsias.com
TEST 31 Solutions

Justification: Statement 1: You can find it here


http://tinyurl.com/z6ed2m8

Statement 2: They were followers of the Mahayana and Tantric schools


of Buddhism. The Pala legacy is still reflected in Tibetan Buddhism. The
Buddhist scholars from the Pala empire travelled from Bengal to other
regions to propagate Buddhism

Statement 3: The Indo-Greeks disappeared as a political entity around


10 AD following the invasions of the Indo-Scythians, although pockets of
Greek populations probably remained for several centuries longer under
the subsequent rule of the Indo-Parthians and Kushans.

Q Source: NCERT 12th History + Past year papers UPSC

18. Consider the following matches of places of Archaeological


Monuments with the states they are found in.
1. Sisupalgarh : Odisha
2. Bishnupur : West Bengal
3. Goalpara : Rajasthan
4. Piprahwa : Uttar Pradesh

Select the correct answer using the codes below.

a) 1, 2 and 4 only
b) 1 and 3 only
c) 2 only
d) 1, 2, 3 and 4

Solution: a)
Justification: Statement 1: Sisupalgarh are a ruined fortification in
Khurda District in Odisha, India. It is one of the largest and best
preserved early historic fortifications in India.
www.insightsonindia.com
www.insightsias.com
TEST 31 Solutions

Statement 2: Bishnupur located in West Bengal is famous for its


terracotta temples Bishnupuriya Raslilla and the balucheri sarees.

Statement 3: Goalpara is located on the bank of the river Brahmaputra.


Goalpara Kingdom was being ruled by Koch Rajbongshi Kings.

Statement 4: Piprahwa is a village near Birdpur in Siddharthnagar


district of Uttar Pradesh. Kalanamak, a scented and spicy variety of rice
is grown in this area. Piprahwa is best known for its archaeological site.

Q Source: Past year UPSC papers

19.Spirulina, a cyanobacteria, is being popularized as a rich source


of some nutrients. It is rich in
a) Folic Acid
b) Proteins
c) Carbohydrates
d) Fats

Solution: b)
Learning: Spirulina is a type of blue-green algae that is rich in protein,
vitamins, minerals, carotenoids, and antioxidants that can help protect
cells from damage.

It contains nutrients, including B complex vitamins, beta-carotene,


vitamin E, manganese, zinc, copper, iron, selenium, and gamma
linolenic acid (an essential fatty acid).

For every 100g of Spirulina consumed, it contains 57g protein.

It also detoxes Heavy Metals (especially Arsenic).

It also lowers blood pressure and reduces cholesterol.

Q Source: Improvisation: Past year UPSC papers


www.insightsonindia.com
www.insightsias.com
TEST 31 Solutions

20. Rabindranath Tagore renounced his knighthood in response


to
a) Jallianwalla Bagh incident
b) Cruelty on Indian National Congress (INC) leaders during
the Simon Commission protests
c) Council Entry movement of Swarajists
d) Partition of Bengal

Solution: a)
Learning: Rabindranath was conferred Knighthood in 1915, the date of
birth of 5th George in recognition of the name he has established in
India and Europe and of his genius as a poet.

Rabindranath Tagore discarded this Title of Knighthood for the inhuman


act of British Government in Jallianwalabag in 1919. Gopal Krishna
Gokhale refused to accept the same title on political reason.

The text of Tagore can be found here

http://timesofindia.indiatimes.com/india/Tagore-renounced-his-
Knighthood-in-protest-for-Jalianwalla-Bagh-mass-
killing/articleshow/7967616.cms

He was shocked at the extreme Brutality shown by the Britishers in the


massacre.

Q Source: 12th TamilNadu History Textbook

21.Consider the following statements about High Courts in India.

Assertion (A): The State governments have established a High Court in


every State and Union territory (UT) of India.
www.insightsonindia.com
www.insightsias.com
TEST 31 Solutions

Reason (R): The Constitution of India empowers only the State


governments to create High Courts.

In the context of the above, which of these is correct?

a) A is correct, and R is an appropriate explanation of A.


b) A is correct, but R is not an appropriate explanation of A.
c) A is correct, but R is incorrect.
d) Both A and R are incorrect.

Solution: d)
Justification: The Centre recently in 2013 constituted three new High
Courts in the northeast Meghalaya, Manipur and Tripura. So, clearly R
is wrong.

But, every state does not have a high court. So, A is also wrong.

You can find here the list of High courts and the act/order that
established them.

https://en.wikipedia.org/wiki/List_of_High_Courts_of_India#High_C
ourts

Q Source: Indian Polity: M Laxmikanth

22. Consider the following statements.

Assertion (A): Balance of Payments (BoP) is a better indicator of a


nations external economic transactions than the trade balance.

Reason (R): BoP takes both capital and current accounts into
consideration which includes trade balance as a component.

In the context of the above, which of these is correct?

a) A is correct, and R is an appropriate explanation of A.


b) A is correct, but R is not an appropriate explanation of A.
c) A is correct, but R is incorrect.
d) Both A and R are incorrect.

Solution: a)
www.insightsonindia.com
www.insightsias.com
TEST 31 Solutions

Justification: BoP encompasses all transactions between a countrys


residents and its non-residents involving goods, services and income;
financial claims on and liabilities to the rest of the world; and transfers
such as gifts.

The balance of payments classifies these transactions in two accounts


the current account and the capital account.

The current account includes transactions in goods, services, investment


income and current transfers, while the capital account mainly includes
transactions in financial instruments.

Q Source: Indian Economy: Ramesh Singh

23. The Direct to Home Service (DTH) television broadcast that


you access at your home uses which of the following satellites?
a) Polar Satellites
b) Geostationary Satellite
c) Low Earth Orbit Satellite
d) Sun-synchronous satellites

Solution: b)
Justification: Satellite television is a system of delivering television
programming using signals relayed from communication satellites which
orbit around the earth at 35,786 KM above the equator in geostationary
orbits.

Tata-Sky a digital cable service provider is associated with DTH


television broadcasting services from INSAT-4A.

Learning: Geostationary satellites in orbits circle the Earth at the same


rate as the Earth spins.

The Earth actually takes 23 hours, 56 minutes, and 4.09 seconds to


make one full revolution.
www.insightsonindia.com
www.insightsias.com
TEST 31 Solutions

So based on Kepler's Laws of Planetary Motion, this would put the


satellite at approximately 35,790 km above the Earth.
The satellites are located near the equator since at this latitude,
there is a constant force of gravity from all directions. At other
latitudes, the bulge at the centre of the Earth would pull on the
satellite.
Geosynchronous orbits allow the satellite to observe almost a full
hemisphere of the Earth.
These satellites are used to study large scale phenomenon such as
hurricanes, or cyclones. These orbits are also used for
communication satellites.
The disadvantage of this type of orbit is that since these satellites
are very far away, they have poor resolution. The other
disadvantage is that these satellites have trouble monitoring
activities near the poles.

You can see this link to understand the difference between major orbit
types
https://marine.rutgers.edu/cool/education/class/paul/orbits2.html

Q Source: Often in news

24. Knowledge of Samudragupta's military exploits comes from


the
a) Allahabad Pillar of Ashoka which includes a eulogy extolling
the deeds and virtues of the Gupta emperor.
b) Sangam literature that mentions the exploits of royal
kingdoms of Northern India
c) Understanding of encrypted carvings at temples and
sculptures constructed after every major military expedition
of Samudragupta
d) Samudravaqta composed by the Chieftians and historians of
the royal court during his reign

Solution: a)
www.insightsonindia.com
www.insightsias.com
TEST 31 Solutions

Learning: Ashoka pillars are series of pillars or columns which are


spread all over the northern Indian sub- continent region. These pillars
signify the Mauryan Empire.

However, from various pillars constructed, currently only few of


them exist.
Allahabad or popularly known as Prayag Raj has one of the famous
Ashoka Pillars.
While it is one of the few extant pillars that carry his edicts, it is
particularly notable for containing later inscriptions attributed to
the Gupta emperor, Samudragupta (4th century CE).
Also engraved on the stone are inscriptions by the Mughal
emperor, Jahangir, from the 17th century
Going by the inscription, Samudragupta exerted direct or indirect
control over much of the Indian subcontinent stretching from
kingdoms in Nepal and the Punjab in the north all the way to the
Pallava kingdom at Kanchipuram in the south-east.

Q Source: Improvisation: UPSC past year papers

25. Consider the following statements.


1. Rajasthan is the largest and Goa is smallest sized state of India.
2. Uttar Pradesh has the largest population followed by
Maharashtra.
3. Leh is the largest sized district of India.
4. The Union Territory (UT) with the highest literacy is
Lakshadweep.

Select the correct answer using the codes below.

a) 1, 2 and 4 only
b) 2, 3 and 4 only
c) 1 and 3 only
d) 1, 2, 3 and 4

Solution: a)
www.insightsonindia.com
www.insightsias.com
TEST 31 Solutions

Justification: Statement 1: Rajasthan is followed by Madhya Pradesh


and Maharashtra. Madhya Pradesh is larger than Maharashtra by a
narrow margin of 532 sq km. Andhra Pradesh (undiv.) is at 4th and
Uttar Pradesh is at 5th place.

Statement 2:
https://en.wikipedia.org/wiki/List_of_states_and_union_territories_of
_India_by_population

Statement 3: 5 largest districts are : kachchh (gujarat - 45652), Leh


(Jammu & kashmir - 45110), Jaisalmer (Rajasthan - 39313), Bikaner
(Rajasthan - 28466), Barmar (Rajasthan - 28393).

Statement 4: Among the union territories size, Andaman and Nicobar


Islands (8,249) ranks first whereas Lakshadweep with area of 60 square
km occupies the lowest rank. 5 union territories have area less than
1,000 sq km.

But, for literacy rates visit


https://en.wikipedia.org/wiki/Indian_union_territories_ranking_by_li
teracy_rate

Q Source: Physical and Human Geography of India

26. Simpson index, Shannon-Wiener index and Alpha diversity


are related to the calculation of
a) Poverty alleviation in selected socio-economic groups
b) Banking competitiveness and portfolio of an economy
c) Biodiversity
d) Occurrence of floods and drought

Solution: c)
Learning: Biodiversity is usually plotted as taxonomic richness of a
geographic area, with some reference to a temporal scale. Ways to
measure biodiversity:
www.insightsonindia.com
www.insightsias.com
TEST 31 Solutions

Species richness - the least sophisticated of the indices available-


the total number of different organisms present. It does not take
into account the proportion and distribution of each species within
the local aquatic community.

Simpson index and Shannon-Wiener index


http://ww2.mdsg.umd.edu/interactive_lessons/biofilm/diverse.ht
m

There are three other indices which are used by ecologists:

Alpha diversity refers to diversity within a particular area,


community or ecosystem, and is measured by counting the number
of taxa within the ecosystem (usually species)

Beta diversity is species diversity between ecosystems; this


involves comparing the number of taxa that are unique to each of
the ecosystems.

Gamma diversity is a measurement of the overall diversity for


different ecosystems within a region.

Q Source: Agritech Website + 12th Biology NCERT

27. Consider the following statements.


1. A High Court judge is appointed by the Governor of the State
concerned who also administers oath to the judge.
2. A person who has acted as a High Court Judge cannot plead
before any authority in India after retirement.

Which of the above is/are correct?

a) 1 only
b) 2 only
c) Both 1 and 2
d) None

Solution: d)
www.insightsonindia.com
www.insightsias.com
TEST 31 Solutions

Justification: Statement 1: President appoints a High Court judge in


consultation with the Governor and Chief Justice of the High Court.

Every person appointed to be a Judge of a High Court shall, before he


enters upon his office, make and subscribe before the Governor of the
State, or some person appointed in that behalf by him an oath or
affirmation according to the form set out for the purpose in the Third
Schedule.

Statement 2: As per the constitution, No person who, after


commencement of this Constitution, has held office as a permanent
judge of High Court shall plead or act in any court or before any
authority except the Supreme Court and the other High Courts.

So, 2 is incorrect as he can plead before other courts except the one
where he held charge.

Q Source: http://archive.indianexpress.com/news/petition-against-
former-judges-practising-in-hc/961243/

28. The thickness of the troposphere varies around the planet.


The troposphere is thicker at the equator than at the poles. Which
of the following factors contribute to it?
1. Higher solar insolation and strong convection currents in
troposphere over the Equator
2. Rotation of the earth

Which of the above is/are correct?

a) 1 only
b) 2 only
c) Both 1 and 2
d) None

Solution: c)
www.insightsonindia.com
www.insightsias.com
TEST 31 Solutions

Justification: The troposphere is thicker over the equator than the


poles because the equator is warmer. The convection currents of air
expand the thickness of the troposphere (atmosphere) at poles.

Thus the simple reason is thermal expansion of the atmosphere at the


equator and thermal contraction near the poles.

Also, the rotation of the earth causes centrifugal force which is strongest
near the equator and pushes the atmosphere to greater heights.

So, both statements 1 and 2 are correct.

Learning: The thickness of the troposphere also varies with season. The
troposphere is thicker in the summer and thinner in the winter all
around the planet. At the poles in winter, the atmosphere is uniformly
very cold and the troposphere cannot be distinguished from other layers.

Q Source: Improvisation: NCERT 11th Fundamentals of Physical


Geography

29. Consider the following statements about Swami Vivekanada.


1. He preached Vedantic philosophy but himself never took the
vow of Sanyasa.
2. He condemned the caste system and unnecessary emphasis on
rituals and ceremonies.
3. He participated at the Parliament of Religions held in Chicago
(USA).

Select the correct answer using the codes below.

a) 1 and 2 only
b) 3 only
c) 2 and 3 only
d) 1, 2 and 3

Solution: c)
www.insightsonindia.com
www.insightsias.com
TEST 31 Solutions

Justification: Statement 1: He became the most famous disciple of


Shri Ramkrishna Paramahamsa and took the vow of Sanyasa. So, 1 is
incorrect.

Statement 2: Condeming the caste system, Vivekananda preached the


message of strength and self-reliance. He asked the people to improve
the lives of the poor and depressed classes. He believed that service to
mankind is service to God.

Statement 3: Swami Vivekananda participated at the Parliament of


Religions held in Chicago (USA) in September 1893 and raised the
prestige of India and Hinduism very high.

He founded the Ramkrishna Mission at Belur in Howrah in 1897. It is a


social service and charitable society. The objectives of this Mission are
providing humanitarian relief and social work through the establishment
of schools, colleges, hospitals and orphanages.

Q Source: 12th TamilNadu History Textbook

30. What is the purpose behind awarding India Handloom tags


by the government?
a) To create a brand for promoting traditional hand woven
heritage of India
b) To allow only certain category of India handicrafts to be
exported abroad
c) To protect selected tribal handloom mills in remote areas of
the country
d) To reduce dependence on powerlooms in view of the
employment generation potential of handlooms

Solution: a)
Learning: The India Handloom Brand (IHB) was launched by the
Prime Minister of India on the occasion of first handloom day in August
2015 to endorse the quality of the products in terms of raw material,
www.insightsonindia.com
www.insightsias.com
TEST 31 Solutions

processing, weaving and other parameters besides social and


environmental compliances for earning the trust of the customers.

The India Handloom brand would be given only to high quality defect
free authentic handloom products for catering to the needs of those
consumers who are looking for niche handmade products. The India
Handloom Brand is aimed at generating a special market space and
increased earnings to the weavers.

Q Source: http://www.textilescommittee.gov.in/india-handloom
http://www.indiahandloombrand.gov.in/pages/know-your-products

31. Which of the following national parks derives its name from the
abundance of Semul (red silk cotton trees) that bloom abundantly
here?
a) Simlipal National Park
b) Kaziranga National Park
c) Mullaperiyar National Park
d) Neora Valley National Park

Solution: a)
Learning: Simlipal National Park is also a tiger reserve situated in the
Mayurbhanj district of Odisha.

It is part of the Similipal-Kuldiha-Hadgarh Elephant Reserve


popularly known as Mayurbhanj Elephant Reserve, which includes
three protected areas Similipal Tiger Reserve, Hadgarh Wildlife
Sanctuary and Kuldiha Wildlife Sanctuary.
Simlipal National Park derives its name from the abundance of
semul (red silk cotton trees) that bloom abundantly here.
o Silk cotton tree is a type of native cotton tree with large red
flowers.
o These are full of cotton-like fibrous stuff. It is for the fiber
that villagers gather the semul fruit and extract the cotton
www.insightsonindia.com
www.insightsias.com
TEST 31 Solutions

substance called "kopak". This substance is used for filling


economically priced pillows, quilts, sofas etc. The fruit is
cooked and eaten and also pickled.
o The tree is also used for wooden work.
Besides Simlipal is famous for gaurs (Indian bison), chausingha as
well as an orchidarium. It has some beautiful waterfalls like
Joranda and Barehipani.
Simlipal is home to royal Bengal tigers and wild elephants.

Q Source: National parks

32. Why was Simon Commission (1927) opposed in India?


a) It was setup to review constitutional safeguards in India.
b) There was no timeline setup for the implementation of
recommendations of the Commission.
c) British unnecessarily dragged India in the First World War.
d) There was no Indian member in the Commission.

Solution: d)
Learning: A standard question.

The Act of 1919 included a provision for its review after a lapse of ten
years.

However, the review commission was appointed by the British


Government two years earlier of its schedule in 1927.

It came to be known as Simon Commission after the name of its


chairman, Sir John Simon. All its seven members were
Englishmen.

As there was no Indian member in it, the Commission faced a lot of


criticism even before its landing in India. Almost all the political
parties including the Congress decided to oppose the Commission.

On 3 February 1928 when the Commission reached Bombay, a


general hartal was observed all over the country.
www.insightsonindia.com
www.insightsias.com
TEST 31 Solutions

Everywhere it was greeted with black flags and the cries of Simon
go back. At Lahore, the students took out a large anti-Simon
Commission demonstration on 30 October 1928 under the
leadership of Lala Lajpat Rai.

In this demonstration, Lala Lajpat Rai was seriously injured in the


police lathi charge and he passed away after one month.

Q Source: 12th TamilNadu History Textbook

33. The Deen dayal Antoydaya Yojana focuses on


1. Promotion of self-employment for poor through setting up
individual micro-enterprises
2. Enabling urban poor form Self-Help Groups for meeting
financial and social needs

Which of the above is/are correct?

a) 1 only
b) 2 only
c) Both 1 and 2
d) None

Solution: c)
Justification: It is an overarching scheme for uplift of urban and rural
poor through enhancement of livelihood opportunities through skill
development and other means.

NRLM (Aajevika mission) was renamed as DAY.

Under the urban component of DAY, focus will be on:

Imparting skills for each urban poor;

Promotion of self-employment through setting up individual


micro-enterprises and group enterprises with interest subsidy for
individual projects
www.insightsonindia.com
www.insightsias.com
TEST 31 Solutions

Training urban poor to meet the huge demand from urban citizens
by imparting market oriented skills through City Livelihood
Centres.

Enabling urban poor form Self-Help Groups for meeting financial


and social needs

Development of vendor markets besides promotion of skills of


vendors; and

Construction of permanent shelters for urban homeless and


provision of other essential services.

Q Source: http://aajeevika.gov.in/content/welcome-deen-dayal-
antyodaya-yojana-nrlm

34. Which of the following characterizes Horse latitudes?


a) High pressure zone characterized by calm winds and little
precipitation
b) Cold frigid environment resulting in cold ocean waves and
currents
c) An area at equator that is the meeting point of trade winds
from north and south
d) Low pressure zone receiving high precipitation

Solution: a)
Justification: The horse latitudes are located at about 30 degrees north
and south of the equator.

It is common in this region of the subtropics for winds to diverge and


either flow toward the poles (known as the prevailing westerlies) or
toward the equator (known as the trade winds).

These diverging winds are the result of an area of high pressure,


which is characterized by calm winds, sunny skies and little or no
precipitation.
www.insightsonindia.com
www.insightsias.com
TEST 31 Solutions

According to legend, the term comes from ships sailing to the New
World that would often become stalled for days or even weeks
when they encountered areas of high pressure and calm winds.
Many of these ships carried horses to the Americas as part of their
cargo. Unable to sail and resupply due to lack of wind, crews often
ran out of drinking water. To conserve scarce water, sailors on
these ships would sometimes throw the horses they were
transporting overboard. Thus, the phrase 'horse latitudes' was
born.

Q Source: Improvisation: NCERT 7th Geography

35. Consider the following matches of monastery with states they


are located in.
1. Dhankar Monastery : Assam
2. Rumtek Monastery : Sikkim
3. Tabo Monastery : Himachal Pradesh
4. Kye Monastery : Arunachal Pradesh

Select the correct answer using the codes below.

a) 1 and 4 only
b) 2 and 3 only
c) 1, 2 and 4 only
d) 3 and 4 only

Solution: b)
Justification: Statement 1: Dhankar Gompa is located in the district of
Lahaul and Spiti in India at an elevation of nearly 3,900 metres between
the towns of Kaza and Tabo.

The complex is built on a 1000-foot (300-metre) high spur overlooking


the confluence of the Spiti and Pin Rivers - one of the world's most
spectacular settings for a gompa.
www.insightsonindia.com
www.insightsias.com
TEST 31 Solutions

Statement 2: Rumtek Monastery, also called the Dharmachakra Centre,


is located in Sikkim near Gangtok. It is a focal point for the sectarian
tensions within the Karma Kagyu school of Tibetan Buddhism that
characterize the Karmapa controversy.

Statement 3: Tabo is located in Spiti Valley, Himachal Pradesh. It was


founded in 996 CE in the Tibetan year of the Fire Ape.

Tabo is noted for being the oldest continuously operating Buddhist


enclave in both India and the Himalayas.

Statement 4: Kye Gompa is a Tibetan Buddhist monastery located on


top of a hill at an altitude of about 4,000 metres above sea level, close to
the Spiti River, in the Spiti Valley of Himachal Pradesh

Q Source: UPSC past year papers

36. Dugong, a herbivorous mammal, which is depleting fast in


number in India is found in
1. Gulf of Mannar
2. Gulf of Kutch
3. Andaman and Nicobar Islands

Select the correct answer using the codes below.

a) 1 and 2 only
b) 2 and 3 only
c) 1 only
d) 1, 2 and 3

Solution: d)
Learning: A highly isolated breeding population exists in the Marine
National Park, Gulf of Kutch, the only population remaining in western
India.
www.insightsonindia.com
www.insightsias.com
TEST 31 Solutions

A population exists in the Gulf of Mannar Marine National Park and the
Palk Strait between India and Sri Lanka, but it is seriously depleted.

Once distributed throughout the coastal belt in Sri Lanka, the


dugong number declined in last two decades due to heavy hunting
by the fishermen.

There are just 250 dugongs in the Indian seas, according to a study
by Zoological Survey of India in 2013.

Conservation in other places like Australia has seen their


population crossing 85,000.

In the Gulf of Mannar, the dugong population ranged between 77


and 158, as per the survey. In Andamans, there could be 41 to 81
dugongs, and in the Gulf of Kutch, some 10 to 15, as per the ZSI
director.

Q Source:
http://timesofindia.indiatimes.com/home/environment/flora-
fauna/Dugongs-on-brink-of-extinction-poaching-continues-in-
India/articleshow/48037397.cms

37. Majhgawan located in Panna is famous for


a) Diamond Mines
b) Uranium mines
c) Gold mines
d) Silver mines

Solution: a)
Learning: India was the only source of diamonds in the world for over
3000 years till diamond deposits were located in Brazil and South Africa.

Madhya Pradesh has a proved and probable diamond reserve of


about 10,45,000 carats. Panna has a estimated total reserve of
about 976 thousand carats.
www.insightsonindia.com
www.insightsias.com
TEST 31 Solutions

Today, NMDC is the only organised producer of diamond in India


from Majhgawan mine at Panna.

Another important mining location in MP is Balaghat district.

Balaghat district of Madhya Pradesh is famous for its Manganese


ore mines and Copper mines.

The district is very rich in forest wealth. About 52% of the area is
covered with forest.

About 80% of the Manganese production of the country comes


from Balaghat. The recent discovery of Copper deposit at
Malajkhand is regarded as the largest in the country.

Q Source: Improvisation: UPSC past year papers

38. Consider the following about World Conservation


Monitoring Centre (WCMC).
1. It is an agency of the United Nations Environment Programme
(UNEP).
2. It is responsible for assessment of biodiversity and contributes
to IUCN Red List.

Which of the above is/are correct?

a) 1 only
b) 2 only
c) Both 1 and 2
d) None

Solution: c)
Justification: Statement 1: The World Conservation Monitoring
Centre was previously an independent organization jointly managed by
www.insightsonindia.com
www.insightsias.com
TEST 31 Solutions

IUCN, UNEP and WWF established in 1988, and prior to that the Centre
was a part of the IUCN Secretariat.

UNEP-WCMC has been part of UNEP since 2000 as its executive agency.

Statement 2: It manages the World Database of Protected Areas in


collaboration with the IUCN World Commission on Protected Areas.

It also supports international conventions such as the Convention


on Biological Diversity (CBD) and CITES, capacity building and
management of both aspatial and spatial data on species and
habitats of conservation concern.
UNEP-WCMC has a mandate to facilitate the delivery of the global
indicators under the CBD's 2010 Biodiversity Target on the rate of
loss of biological diversity, and works alongside the CITES
Secretariat producing a range of reports and databases.

Q Source: UNEP Website

39. Which of the following factors does NOT cause disguised


unemployment in India?
a) Low skill base of labour
b) Large population amidst high rates of unemployment
c) Poor on-field production management
d) Government intervention in the economy

Solution: d)
Justification & Learning: In India, this is more common in
agriculture and low productivity sectors like small households
establishments. Most industrial units in India face the issue of disguised
unemployment.

Option (a): If the labour has a low skill base, he cannot contribute
much to production and addition of further labour has little or no
impact on the total production.
www.insightsonindia.com
www.insightsias.com
TEST 31 Solutions

Option (b): Large population, low level of capital utilization and


technology aggravate this situation amidst already high
unemployment.
Option (c): Finally, due to poor management of workers, some
sites remain overemployed causing poor labour productivity.

Government intervention will exist in one or the other form in an


economy and instead brings down the level of unemployment, generates
more jobs and cuts down unhealthy labour practices. So, (d) clearly is the
answer.

Q Source: 11th NCERT: Indian Economy

40. Shimla is cooler than Amritsar even though they are located
on the same latitude. This is because
a) Amritsar is influenced by warm westerly winds in winter.
b) Shimla is at a greater altitude than Amritsar.
c) Shimla is located at a different longitude.
d) Amritsar is closer to the Sea than Shimla.

Solution: b)
Justification: Temperatures drop the further an area is from the
equator due to the curvature of the earth.

Also, locations at a higher altitude have colder temperatures.


Temperature usually decreases by 1C for every 100 metres in altitude.

Shimla is a hill station with an average altitude of 2,206 metres (7,238 ft)
above mean sea level, which is much greater than Amritsar, hence the
lack of warmth.

Option (c) is incorrect as longitude doesnt affect climate as long as other


factors do not change.

Q Source: 9th Geography NCERT


www.insightsonindia.com
www.insightsias.com
TEST 31 Solutions

41.The Gulf Cooperation Council (GCC) is a/an


a) Intergovernmental political and economic union of Arab
states
b) Trade cartel that regulates oil prices in international markets
c) Informal grouping of all West Asian nations for promoting
cultural contact
d) Community of immigrant labour working in Gulf countries
aimed at improving work conditions in the Middle-east

Solution: a)
Learning: It consists of all Arab states of the Persian Gulf, except for
Iraq.

Its member states are Bahrain, Kuwait, Oman, Qatar, Saudi


Arabia, and the United Arab Emirates.
All current member states are monarchies.
A 2011 proposal to transform the GCC into a "Gulf Union" with
tighter economic, political and military coordination has been
advanced by Saudi Arabia, a move meant to counterbalance the
Iranian influence in the region.
Objections have been raised against the proposal by other
countries.
A common market was launched on 1 January 2008 with plans to
realise a fully integrated single market. A nominal GCC single
currency is also used to some extent.
GCC is a major trade partner of India.

Q Source: http://www.mea.gov.in/regional-organisations.htm
www.insightsonindia.com
www.insightsias.com
TEST 31 Solutions

42. These regions are hot and receive heavy rainfall throughout
the year. As there is no particular dry season, the trees do not shed
their leaves altogether. This description matches
a) Temperate Decidous forests
b) Tropical Evergreen Forests
c) Mediterranean Vegetation
d) Monsoon forests

Solution: b)
Learning: Forests found near the Equator (10 North to 10 South) in
Amazon & Zaire Basin and Southeast Asia are major evergreen forests.

The thick canopies of the closely spaced trees do not allow the
sunlight to penetrate inside the forest even in the day time. Thus,
grass is not found in these forests.
Maximum varieties of trees are found. Trees are tall with large
trunks.
Hardwood trees like rosewood, teak, sal, ebony, and mahogany are
the common trees found here.
Here the population found is very less. Due to dense forests
commercial exploitation of these forests has not be possible,
making them economically backward.

Q Source: 8th NCERT Geography

43. Which of the following is/are member(s) of the East Asia


Summit?
1. India
2. Bangladesh
3. Sri Lanka
4. China
5. USA

Select the correct answer using the codes below.


www.insightsonindia.com
www.insightsias.com
TEST 31 Solutions

a) 3, 4 and 5 only
b) 2, 3 and 4 only
c) 1, 4 and 5 only
d) 1, 2 and 3 only

Solution: c)
Justification: The way options have been set should lead you directly
to the answer.

It is common knowledge that USA and China both are members of the
East Asia summit, and Sri Lanka is not a member. So, the only correct
option can be (c).

Learning: Established in 2005, EAS allows the principal players in the


Asia-Pacific region to discuss issues of common interest and concern in
an open and transparent manner at the highest level.

The membership of EAS consists of ten ASEAN Member States (i.e.


Brunei Darussalam, Cambodia, Indonesia, Lao PDR, Malaysia,
Myanmar, Singapore, Thailand, the Philippines and Vietnam), Australia,
China, India, Japan, New Zealand, Republic of Korea, Russian
Federation and the USA.

EAS is an initiative of ASEAN and is based on the premise of the


centrality of ASEAN.

Q Source: http://www.mea.gov.in/regional-organisations.htm

44. Polavaram Project is a multi-purpose irrigation project which


has been accorded national project status by the central
government. How will the project be helpful?
a) It will replace all canals in Southern India by a single channel
for improved water delivery to fields.
b) It will stop floods in Northern India.
www.insightsonindia.com
www.insightsias.com
TEST 31 Solutions

c) It will lead to transfer of water from surplus Godavari basin


to the water deficit Krishna basin.
d) It will reduce wastage of riverine lands now under Delta.

Solution: c)
Learning: This dam across the Godavari River is under construction
located in West Godavari District and East Godavari District in
Andhra Pradesh state and its reservoir spreads in parts of
Chhattisgarh and Orissa States also.

It is for development of Irrigation, Hydropower and drinking


water facilities to East Godavari, Vishakhapatnam, West
Godavari and Krishna districts of Andhra Pradesh.
The project is being delayed as it is likely to displace over 1.88
lakh people across 222 villages.
The project implements Godavari-Krishna link under Interlinking
of rivers project. The project envisages transfer of 80TMC of
surplus Godavari water to river Krishna which will be shared
between AP, Karnataka and Maharashtra in proportion of 45 TMC
by AP and 35 TMC by Karnataka and Maharashtra as per the
decision of the GWDT award.

Q Source: http://wrmin.nic.in/forms/list.aspx?lid=380

45. Palmyra temple, Syria, was recently in news due to


www.insightsonindia.com
www.insightsias.com
TEST 31 Solutions

a) Its ancient remains were dug out by an expert team from


India.
b) Being at the Centre of West Bank dispute
c) Damage suffered to important sites by extremist groups
d) UNESCO awarding it status of a World Heritage Site in 2016

Solution: c)
Learning: Archaeological finds of Palmyra date back to the Neolithic
period, and the city was first documented in the early second millennium
BC. So, (a) is incorrect.

It was already a World Heritage site (UNESCO). So, (d) is incorrect.

In 2015, Palmyra came under the control of the Islamic State of


Iraq and the Levant (ISIL), which later destroyed a number of the
site's buildings. The city was retaken by the Syrian Army.
Palmyra is an ancient Semitic city in present-day Homs
Governorate, Syria.
Palmyra features Roman colonnades and an impressive necropolis.
Much of Palmyra is well-preserved the uniqueness of the city,
an oasis on the Silk Road, stems from the fact that so much
remains intact from Roman times, including the remains of
public buildings, temples and administrative centres.
Palmyra changed hands on a number of occasions between
different empires before becoming a subject of the Roman Empire
in the first century AD.

Q Source: In news

46. The suitable areas for aerobic rice cultivation are those where
a) Rainfall is high enough to submerge the entire crop
b) Soil does not has any nutrients
c) Hard salt pans have formed in the soil pores
d) Rainfall is insufficient to sustain rice production
www.insightsonindia.com
www.insightsias.com
TEST 31 Solutions

Solution: d)
Concept: Aerobic rice cultivation is growing rice plant as irrigated crop
like cultivating maize and wheat in aerobic condition, where oxygen is
plenty in soil.

Throughout the growing season, aerobic rice field is kept under


unsaturated condition and field is irrigated by surface or sprinkler
system to keep soil wet. Therefore, water productivity is reported
to be higher in aerobic rice.
In aerobic rice cultivation, rice is cultivated as direct sown in
nonpuddle aerobic soil under supplementary irrigation and
fertiliser with suitable high yielding rice varieties.

Justification: The suitable areas for aerobic rice cultivation includes


irrigated lowlands, where rainfall is insufficient to sustain rice
production, delta regions where there is delay in water release from
reservoir, irrigated system of rice cultivation, where pumping from deep
bore well has become so expensive and favourable upland system has
access to supplementary irrigation.

Accordingly, Tamil Nadu, Jharkhand, Chhattisgarh, parts of


Bihar, Odisha, Karnataka, and eastern Uttar Pradesh are the
projected area where there is uneven distribution and frequent
occurrence of soil moisture limitation.
Aerobic rice cultivation needs suitable rice varieties having the
characteristics of both upland and high yielding lowland
varieties to get good yield under the new unconventional
system of cultivation.
This system also involves mechanised way of sowing with no
puddling, transplanting and no need of frequent irrigation,
which reduce labour usage more than 50%, compared to
irrigated rice.

Q Source: http://irri.org/our-work/research/better-rice-
varieties/aerobic-rice
www.insightsonindia.com
www.insightsias.com
TEST 31 Solutions

47. The President of India can proclaim National Emergency


under which of the following conditions?
a) Report of rampant corruption in States
b) Armed invasion by another nation in India
c) Loss of lives due to a natural calamity
d) Failure to form a government at the Centre

Solution: b)
Learning: National emergency can be declared under A352 on the basis
of external aggression or armed rebellion in the whole of India or a part
of its territory. Such an emergency was declared in India in 1962 (Indo-
China war), 1971 (Indo-Pakistan war), and 1975 (declared by Indira
Gandhi).

The President can declare such an emergency only on the basis of a


written request by the Council of Ministers headed by the Prime
Minister. Such a proclamation must be laid before both houses of
Parliament, and the state of emergency expires after one month
unless approved within that time by both houses sitting and voting
separately.
It is not declared in case of epidemics, natural calamities or failure
to form government at the Centre which makes all options other
than (b) wrong.

Q Source: Indian Polity: M Laxmikanth

48. A type of food poisoning called botulism is caused by a


bacterium due to a toxin produced by it. This toxin is extensively
used in
www.insightsonindia.com
www.insightsias.com
TEST 31 Solutions

a) Reducing pain during surgery


b) Improving the milk yield of cattle
c) In treating mouth cancer
d) In removing facial wrinkles

Solution: d)
Learning: Botulism is a rare but potentially life-threatening bacterial
illness.

Botox is a drug made from a neurotoxin produced by the bacterium


Clostridium botulinum called botulinum toxin. It is used medically to
treat certain muscular conditions and cosmetically remove wrinkles by
temporarily paralyzing muscles.

Clostridium Botulinum bacteria grows on food and produces toxins


that, when ingested, cause paralysis. Botulism poisoning is
extremely rare, but so dangerous that each case is considered a
public health emergency.
C. botulinum is a soil bacterium. The spores can survive in most
environments and are very hard to kill.
They can survive the temperature of boiling water at sea level, thus
many foods are canned with a pressurized boil that achieves even
higher temperatures, sufficient to kill the spores.

Q Source: UPSC past year papers

49. Consider the following statements about fundamental forces


of nature.
1. Gravitational force is the strongest of all forces due to its large
range.
2. Electromagnetic force is the weakest force due to its extremely
short range of influence.

Which of the above is/are correct?


www.insightsonindia.com
www.insightsias.com
TEST 31 Solutions

a) 1 only
b) 2 only
c) Both 1 and 2
d) None

Solution: d)
Justification: Statement 1: Gravitation is by far the weakest of the four
interactions. The weakness of gravity can easily be demonstrated by
suspending a pin using a simple magnet (such as a refrigerator magnet).
The magnet is able to hold the pin against the gravitational pull of the
entire Earth.

Statement 2: Electromagnetism is the force that acts between electrically


charged particles. This phenomenon includes the electrostatic force
acting between charged particles at rest, and the combined effect of
electric and magnetic forces acting between charged particles moving
relative to each other.

Electromagnetism is infinite-ranged like gravity, but vastly stronger, and


therefore describes a number of macroscopic phenomena of everyday
experience such as friction, rainbows, lightning, and all human-made
devices using electric current, such as television, lasers, and computers.

This table is useful to remember information.

Q Source: http://hyperphysics.phy-
astr.gsu.edu/hbase/forces/funfor.html
www.insightsonindia.com
www.insightsias.com
TEST 31 Solutions

50. Dhanush developed by Defence Research and Development


Organisation (DRDO) is a
a) Surface-to-surface missile and a naval variant of Indias
indigenously-developed Prithvi missile
b) Nuclear submarine recently contracted to Russia
c) Amphibian aircraft handed over by Japan to India as a
replacement for Airavat
d) Anti-war ship located in the Southern Indian Ocean to stop
piracy

Solution: a)
Learning: Dhanush is a variant of the surface-to-surface/ship-to-ship
Prithvi III missile, which has been developed for the Indian Navy.

It is capable of carrying both conventional as well as nuclear


warheads with pay-load capacity of 500 kg-1000 kg and can strike
targets in the range of 350 km.

The Missile was test-fired recently in 2015.

The Dhanush missile can be used as an anti-ship weapon as well as


for destroying land targets depending on the range.

The missile gives the Indian Navy the capability to strike enemy
targets with great precision.
www.insightsonindia.com
www.insightsias.com
TEST 31 Solutions

Q Source:
http://www.drdo.gov.in/drdo/English/index.jsp?pg=dhanush.jsp

51. The Centre has decided to develop the Doyang Lake in Nagaland as
an eco-tourism spot for bird-watchers. The lake is famous as a
roosting site for
a) Great Indian Bustard
b) Amur falcons
c) Common Myna
d) White Florican

Solution: b)
Learning: The Centre will develop the Doyang lake in Nagaland -
famous for the world's longest travelling raptors Amur falcons - as an
eco-tourism spot.
www.insightsonindia.com
www.insightsias.com
TEST 31 Solutions

Amur falcons come to Doyang every year in millions. They come to


roost here during their flight from Mongolia to South Africa,
making this beautiful area in India's northeast a bird-watchers'
paradise.

The lake area had come to limelight when two of three falcons,
tagged with satellite device in 2013 to understand their migratory
behaviour, had returned to the lake twice after taking rounds from
Mongolia to South Africa via Nagaland.

These two falcons - named Naga and Pangti - have once again
returned to the lake in Nagaland this year.

As per the Union MoEFCC minister, The world has recognized


Pangti village in Nagaland as the worlds Amur Falcon capital, as
more than one million birds can be seen in just 30 minutes.

Until recently, Naga tribesmen used to hunt thousands of


Amur falcons for meat. But last year, after a vigorous
campaign by wildlife activists, they pledged to protect the bird
and since then, not a single bird has been hunted in the area.

Q Source: http://pib.nic.in/newsite/PrintRelease.aspx?relid=130503

52. Marginal Cost of funds based Lending Rate (MCLR), as


announced recently by the RBI, has replaced the
a) Repo rate and CRR system
b) Statutory Liquidity Ratio (SLR) system
c) Base rate system
d) Basel system

Solution: c)
Learning: The Reserve Bank of India has brought a new methodology
of setting lending rate by commercial banks under the name Marginal
Cost of Funds based Lending Rate (MCLR). It has modified the existing
base rate system from April 2016 onwards.
www.insightsonindia.com
www.insightsias.com
TEST 31 Solutions

As per the new guidelines by the RBI, banks have to prepare


Marginal Cost of Funds based Lending Rate (MCLR) which will be
the internal benchmark lending rates. Based upon this MCLR,
interest rate for different types of customers should be fixed in
accordance with their riskiness. The base rate will be now
determined on the basis of the MCLR calculation.

The MCLR should be revised monthly by considering some new


factors including the repo rate and other borrowing rates.
Specifically the repo rate and other borrowing rates that were not
explicitly considered under the base rate system.

The change was brought as: Commercial banks are significantly


depending upon the RBIs LAF repo to get short term funds. But they are
reluctant to change their individual lending rates and deposit rates with
periodic changes in repo rate.

Q Source:
http://www.livemint.com/Money/Xizd6EP6QOnFTl7jtxbQSO/10-
things-to-know-about-the-new-loan-rate.html

53. Given below are species with some of the regions they are
dominantly found in.

(Species) (Region)

1. Shrew and Tapir A. Himalayan range

2. Indian bison B. Thar Deserts

3. Indian Gazelle C. Western Ghats

The correct match for the above is?

a) 1-A, 2-C, 3-B


b) 1-B, 2-A, 3-C
c) 1-B, 2-C, 3-A
d) 1-A, 2-B, 3-C
www.insightsonindia.com
www.insightsias.com
TEST 31 Solutions

Solution: a)
Learning: The great Himalayan range hosts wild sheep and goats,
markhor, ibex, shrew and tapir. The panda and the snow leopard are
found in the upper reaches of the mountains.

The Western Ghats and their outflanking hills in southern India


constitute one of the most extensive extant strongholds of gaur (Indian
Bison), in particular in the Wayanad Nagarhole Mudumalai
Bandipur complex.

Chinkara live in arid plains and hills, deserts, dry scrub and light forests.
In 2001, the Indian chinkara population was estimated at 100,000 with
80,000 living in the Thar Desert.

Q Source: Species often asked by UPSC

54. Auction of spectrum has been frequently in news. In this


context, Spectrum is related with?
a) Frequency range for wireless communication
b) Amount of data consumed on internet network
c) The total value of power assigned to corporations free of cost
d) Devices that make use of nuclear forces

Solution: a)
Learning: We take the example of Radio waves.

They are a form of electromagnetic radiation which, like visible


light or infrared, make up a portion of the entire spectrum. They
cannot be perceived by human eyes or ears, and they are not
harmful in the environment.

The term radio spectrum typically refers to the full frequency range
from 3 kHz to 300 GHz that may be used for wireless
communication.

In India, the Department of Telecommunications (DoT) conducts


auctions of licenses for electromagnetic spectrum. India was
www.insightsonindia.com
www.insightsias.com
TEST 31 Solutions

among the early adopters of spectrum auctions beginning auctions


in 1991.

The government is planning a mega-spectrum auction likely in


September in which a record quantity of airwaves, including those
in the coveted 700 Mhz band, will be up for the sale.

Q Source:
http://economictimes.indiatimes.com/news/company/corporate-
trends/telenor-says-wont-participate-in-spectrum-sale-signals-india-
exit/articleshow/53280180.cms

55. Bisphenol-A (BPA) is a chemical used in an important


industry that also adversely affects aquatic organisms. How are
humans exposed to it?
a) Nuclear radiation
b) Consuming packaged food
c) Touching surgical instruments
d) Through textile mill workers

Solution: b)
Learning: Bisphenol A (BPA) is a chemical produced in large quantities
for use primarily in the production of polycarbonate plastics and epoxy
resins.

The primary source of exposure to BPA for most people is through


the diet. While air, dust, and water are other possible sources of
exposure, BPA in food and beverages accounts for the majority of
daily human exposure.

Bisphenol A can leach into food from the protective internal epoxy
resin coatings of canned foods and from consumer products such
as polycarbonate tableware, food storage containers, water bottles,
and baby bottles.
www.insightsonindia.com
www.insightsias.com
TEST 31 Solutions

The degree to which BPA leaches from polycarbonate bottles into


liquid may depend more on the temperature of the liquid or bottle,
than the age of the container. BPA can also be found in breast milk.

Evidence of endocrine-related effects in fish, aquatic invertebrates,


amphibians, and reptiles has been reported at environmentally
relevant exposure levels lower than those required for acute
toxicity.

Q Source: Improvisation: Past year UPSC papers

56. Consider the following about food safety logos used in India.
1. Green mark logo is found on the label of pre-packaged
vegetarian food product.
2. India Organic logo certifies that the food product is
organically grown.
3. ISI mark logo should be available on the packaged drinking
water.

Select the correct answer using the codes below.

a) 1 and 2 only
b) 3 only
c) 2 only
d) 1, 2 and 3 only

Solution: d)
Justification: India has a comprehensive system of product
certifications governed by laws.

ISI mark : For industrial product. Certifies that a product conforms


to a set of standards laid by the Bureau of Indian Standards.

FPO mark : A mandatory mark for all processed fruit products in


India. Certifies that the product was manufactured in a hygienic
'food-safe' environment.

Others can be seen in the image below.


www.insightsonindia.com
www.insightsias.com
TEST 31 Solutions

Q Source: Agritech website + UPSC past year papers

57. The north-east monsoon, commonly known as winter


monsoon blows from
a) Sea to land
b) Plains to mountains
www.insightsonindia.com
www.insightsias.com
TEST 31 Solutions

c) Entirely from Upper air circulation


d) Land to Sea

Solution: d)
Learning: The climate of India may be broadly described as tropical
monsoon type.

It is affected by two seasonal winds - the northeast monsoon and


the southwest monsoon.

The north-east monsoon, commonly known as winter monsoon


blows from land to sea, whereas south-west monsoon, known as
summer monsoon blows from sea to land after crossing the Indian
Ocean, the Arabian Sea, and the Bay of Bengal.

The south-west monsoon brings most of the rainfall during a year


in the country.

Q Source: NCERT 9th Geography

58. Tropic of Capricorn and Equator, both, pass through which


of the following countries?
a) Brazil
b) Algeria
c) Egypt
d) Mexico

Solution: a)
Learning: Tropic Of Capricorn passes through following major
countries : Chile , Argentina , Paraguay , Brazil , Botswana , Madagascar
and Australia.

The equator passes through 13 countries: Ecuador, Colombia, Brazil, Sao


Tome & Principe, Gabon, Republic of the Congo, Democratic Republic of
the Congo, Uganda, Kenya, Somalia, Maldives, Indonesia and Kiribati.
www.insightsonindia.com
www.insightsias.com
TEST 31 Solutions

Brazil is common to both lists. The answer is clearly (a).

Q Source: World map based questions

59. The free-rider problem in Economics is associated with


1. Public goods
2. All private goods
3. Foreign Investment

Select the correct answer using the codes below.

a) 1 and 2 only
b) 3 only
c) 2 and 3 only
d) 1 only

Solution: d)
www.insightsonindia.com
www.insightsias.com
TEST 31 Solutions

Justification: The free-rider problem means the Consumers will not


voluntarily pay for what they can get for free and for which there is no
exclusive title to the property being enjoyed.

For e.g. if we consider a public park or measures to reduce air pollution,


the benefits will be available to all irrespective of them paying for it or
not.

The consumption of such products by several individuals is not


rivalrous in the sense that a person can enjoy the benefits without
reducing their availability to others, which results in this free-rider
problem.

In case of private goods anyone who does not pay for the good can
be excluded from enjoying its benefits.

o If you do not buy a ticket, you are excluded from watching a


film at a local theatre. However, in case of public goods, there
is no feasible way of excluding anyone from enjoying the
benefits of the good (they are non-excludable). Since non-
paying users usually cannot be excluded, it becomes difficult
or impossible to collect fees for the public good.

So, clearly 1 is correct and 2 is incorrect. Statement 3 is irrelevant in this


context.

Q Source: 12th NCERT: Macroeconomics

60. Consider the following statements.


1. Excise duty is a direct tax applied only on exports of selected
commodities.
2. Securities Transaction Tax is an indirect tax levied on the
maintenance of industrial and police forces by private
companies.

Which of the above is/are correct?


www.insightsonindia.com
www.insightsias.com
TEST 31 Solutions

a) 1 only
b) 2 only
c) Both 1 and 2
d) None

Solution: d)
Justification: Statement 1: An excise or excise tax is an inland (not on
imports, exports) tax on the sale, or production for sale, of specific goods
or a tax on a good produced for sale, or sold, within a country or licenses
for specific activities.

Statement 2: Securities Transaction Tax (STT) is a tax payable in India


on the value of securities (excluding commodities and currency)
transacted through a recognized stock exchange. The tax is not
applicable on off-market transactions or on commodity or currency
transactions.

Learning: Some other taxes that generally people do not know about
are:

Fringe Benefit Tax (FBT) is fundamentally a tax that an employer


has to pay in lieu of the benefits that are given to his/her
employees. It was an attempt to comprehensively levy tax on those
benefits, which evaded the taxman.

o The list of benefits encompassed a wide range of privileges,


services, facilities or amenities which were directly or
indirectly given by an employer to current or former
employees, be it something simple like telephone
reimbursements, free or concessional tickets or even ..

The Interest Tax Act, 1974 was an Act that imposes a special tax on
interest accrued in specified cases. The Act applied to the whole of
India, including all the States and Union Territories with no
exceptions. The Act is no longer applicable with regards to
chargeable interest accruing after 31 March 2000.

Q Source: Indian Economy: Ramesh Singh


www.insightsonindia.com
www.insightsias.com
TEST 31 Solutions

61.Consider the following statements.


1. The constitution establishes Consolidated Fund of the Union as
well as Consolidated Fund of the States.
2. There is no mention of Public Accounts of India and
Contingency Fund in the constitution as they are established by
legislation.

Which of the above is/are correct?

a) 1 only
b) 2 only
c) Both 1 and 2
d) None

Solution: a)
Justification: Statement 1: Article 266 provides for a Consolidated
Fund of India in which the net proceeds of certain taxes and duties to
States, all revenues received by the Government of India, all loans raised
by that Government by the issue of treasury bills etc. would come.

It also provides for a Consolidated Fund of the States. So, 1 is correct.

Statement 2: The same A266 provides that All other public moneys
received by or on behalf of the Government of India or the Government
of a State shall be credited to the public account of India or the public
account of the State, as the case may be. So, clearly 2 is wrong.

Article 267 says that "Parliament may by law establish a Contingency


Fund to be entitled "the Contingency Fund of India" into which shall be
paid from time to time such sums as may be determined by such law,
and the said Fund shall be placed at the disposal of the President to
enable advances to be made by him out of such Fund for the purposes of
meeting unforeseen expenditure pending authorisation of such
expenditure by Parliament by law under article 115 or article 116.

Q Source: Indian Polity: M Laxmikanth


www.insightsonindia.com
www.insightsias.com
TEST 31 Solutions

62. Mosses that grow in tundra regions are


a) A combination of algae and fungi similar to lichens
b) Green plants that can manufacture their own food
c) A remote species of Fungi
d) An insectivorous plant that feeds on bio-control agents

Solution: b)
Learning: Algae, lichens, and mosses grow profusely on trunks, limbs,
and twigs of many trees and shrubs.

Algae are the simplest of green plants.

Lichens have two componentsa fungus and an alga living in


association with one another to give the appearance of a single
plant.

Mosses are small green flowerless plants that typically grow in


dense green clumps or mats, often in damp or shady locations.

They are somewhat similar to algae except they have a complex


structure that resembles stems and leaves.

Because they contain chlorophyll, mosses can manufacture their


own food. Mosses grow on soils, on tree trunks and branches, on
rocks, and in water. Liverworts are closely related to the mosses
and grow under similar conditions.

Q Source: NCERT 11th: Fundamentals of Physical Geography

63. Fishes migrate on a regular basis, on time scales ranging


from daily to annual. Consider the following about it.
www.insightsonindia.com
www.insightsias.com
TEST 31 Solutions

1. Fish usually migrate because of diet or reproductive needs.


2. Fishes never migrate between fresh water and salt water bodies
due to their bodily limitations.

Which of the above is/are correct?

a) 1 only
b) 2 only
c) Both 1 and 2
d) None

Solution: a)
Justification: Statement 1: For e.g. Forage fish often make great
migrations between their spawning, feeding and nursery grounds.

Schools of a particular stock usually travel in a triangle between


these grounds.

One stock of herrings have their spawning ground in southern


Norway, their feeding ground in Iceland, and their nursery ground
in northern Norway.

Construction of dams, reservoirs etc often affect the migratory


routes of fishes and hence their populations.

Statement 2: In some cases osmotic pressure limits fihs migration


(concept covered in an earlier test), but in other cases it does not.

Potamodromous fish migrate within fresh water only (Greek:


Potamos is river and dromos is 'a running')

Oceanodromous fish migrate within salt water only (Greek:


'Oceanos' is ocean)

Diadromous fish travel between salt and fresh water (Greek: 'Dia'
is between)

Anadromous fish live in the ocean mostly, and breed in fresh water
(Greek: 'Ana' is up; The noun is "anadromy")

Catadromous fish live in fresh water, and breed in the ocean


(Greek: 'Cata' is down)
www.insightsonindia.com
www.insightsias.com
TEST 31 Solutions

Amphidromous fish move between fresh and salt water during


their life cycle, but not to breed (Greek: 'Amphi' is both)

Q Source: Based on UPSC past year papers

64. Sarkaria Commission was concerned with


a) Electoral Reforms
b) Financial Sector Reforms
c) Centre-State relations
d) Reforms in Police administration

Solution: c)
Learning: Sarkaria Commission was set up in June 1983 by the central
government of India.

The Sarkaria Commission's charter was to examine the relationship and


balance of power between state and central governments in the country
and suggest changes within the framework of Constitution of India.

The Commission made some significant recommendations with respect


to the office of the Governor, Centre-state relations like A356
(Presidents rule) etc.

Some of the recommendations can be seen here


https://en.wikipedia.org/wiki/Sarkaria_Commission

Q Source: http://interstatecouncil.nic.in/Sarkaria_Commission.html

65. Pattachitra style of painting is


a) One of the oldest and most popular art forms of Odisha
b) Done on a canvas largely depicting mythological themes
c) Done using natural raw material like from the Kaitha tree
d) All of the above

Solution: d)
www.insightsonindia.com
www.insightsias.com
TEST 31 Solutions

Justification & Learning: The name Pattachitra has evolved from the
Sanskrit words patta, meaning canvas, and chitra, meaning picture.

Pattachitra is thus a painting done on canvas, and is manifested by


rich colourful application, creative motifs and designs, and
portrayal of simple themes, mostly mythological in depiction.

Some of the popular themes represented through this art form are
Thia Badhia - depiction of the temple of Jagannath; Krishna Lila -
enactment of Jagannath as Lord Krishna displaying his powers as a
child; Dasabatara Patti - the ten incarnations of Lord Vishnu etc.

Making the patta is the first thing that comes in the agenda, and
the painters, also called chitrakars, go about their work in
preparing a tamarind paste, which is made by soaking tamarind
seeds in water for three days.

Preparing the paints is perhaps the most important part of the


creation of Pattachitra, engaging the craftsmanship of the
chitrakars in using naturally available raw materials to bring about
indigenous paints. The gum of the kaitha tree is the chief
ingredient, and is used as a base for making different pigments.

Q Source:
http://www.archive.india.gov.in/knowindia/culture_heritage.php?id=1

66. Consider the following statements about health indicators.


1. The infant mortality rate (IMR) takes into account the number
of deaths of infants under one year old.
2. The Maternal mortality rate (MMR) is the total annual number
of female deaths due to child marriage.

Which of the above is/are correct?

a) 1 only
b) 2 only
www.insightsonindia.com
www.insightsias.com
TEST 31 Solutions

c) Both 1 and 2
d) None

Solution: a)
Justification: Statement 1: IMR is calculated per 1,000 live births. A
neonatal death is defined as a death during the first 28 days of life.

Statement 2: MMR is calculated per 100,000 live births from any cause
related to or aggravated by pregnancy or its management.

Both indicators show the status of women and child health in India.

Refer to this article for Indias status and achievements with regards to
IMR and MMR.

http://pib.nic.in/newsite/PrintRelease.aspx?relid=123669

Q Source: Often in news

67. Micronutrients are required for normal growth, development


and yield of crops. Which of the following is NOT a micronutrient
for plants?
a) Zinc
b) Iron
c) Manganese
d) Rubidium

Solution: d)
Learning: There are 7 essential plant nutrient elements defined as
micronutrients [boron (B), zinc (Zn), manganese (Mn), iron (Fe), copper
(Cu), molybdenum (Mo), chlorine (Cl)]. They constitute in total less than
1% of the dry weight of most plants.
www.insightsonindia.com
www.insightsias.com
TEST 31 Solutions

Each nutrient is needed to a particular level for each crop. If any


deficiency or toxicity arises in soil, phonological symptoms will be
exhibited by the crops accordingly.

For example, cotton needs Zn for the normal pollen production.

Fe deficiency will cause chlorosis of younger leaves. Nitrogen,


phosphorus and potassium deficiency symptoms will be first
exhibited in the older leaves.

Boron application to groundnut reduced the occurrence of ill filled


pods and enhances the yield. Boron application to the coconut
reduce the barren nut production.

Pulses need molybdenum for better nodulation.

Manganese application to gingelly increases its oil content an seed


yield. Copper deficiency in citrus causes die-back disease.

Besides the application of needed dose of N,P and K fertilizers, Zn


application increased the yields in rice, maize, soybean and
groundnut.

Q Source: NCERT 8th Science

68. With reference to the United Nations, consider the following


statements.
1. The UN Economic and Social Council (ECOSOC) is elected by
the UN General Assembly.
2. Seats on the ECOSOC are based on geographical representation.

Which of the above is/are correct?

a) 1 only
b) 2 only
c) Both 1 and 2
d) None
www.insightsonindia.com
www.insightsias.com
TEST 31 Solutions

Solution: c)
Justification: It is one of the six principal organs of the United
Nations, responsible for coordinating the economic, social and related
work of 14 UN specialized agencies, their functional commissions and
five regional commissions.

Statement 1: The Council has 54 member states out of the 193 UN


member states, which are elected by the United Nations General
Assembly for overlapping three-year terms

Statement 2: Seats on the Council are based on geographical


representation with 18 allocated to African states, 13 to Asian states, 8 to
East European states, 13 to Latin American and Caribbean states and 13
to West European and other states.

The president is elected for a one-year term and chosen from the small
or mid-sized powers represented on the ECOSOC.

Q Source: UN Website

69. Consider the following statements.


1. Lok Sabha is not automatically dissolved after the end of its
term and continues until new house is not constituted after
elections.
2. The Speaker of Lok Sabha continues in office even after the
dissolution of the House and until immediately before the first
meeting of the House.

Which of the above is/are correct?

a) 1 only
b) 2 only
c) Both 1 and 2
d) None

Solution: b)
www.insightsonindia.com
www.insightsias.com
TEST 31 Solutions

Justification: Statement 1: Unless sooner dissolved or there is an


extension of the term, there is an automatic dissolution of the Lok Sabha
by efflux of time, at the end of the period of five years, even if no formal
order of dissolution is issued by the President.

Statement 2: But, the above does not apply to Speaker, and she
continues in office until the new house is constituted and its first
meeting started.

Q Source: Indian Polity: M Laxmikanth

70. Greenhouse Technology helps to


a) Grow plants under controlled climatic conditions leading to
higher yield
b) Trap all greenhouse gases into a chamber to reduce global
warming
c) Induce cloud seedling and rainfall
d) Capture and sequester electromagnetic waves to control
climatic conditions outside

Solution: a)
Learning: About 95% of plants, either food crops or cash crops are
grown in open field.

But, in some of the temperate regions where the climatic conditions are
extremely adverse and no crops can be grown.

Greenhouse Technology is the technique of providing favourable


environment condition to the plants. It is rather used to protect the
plants from the adverse climatic conditions such as wind, cold,
precipitation, excessive radiation, extreme temperature, insects
and diseases.
www.insightsonindia.com
www.insightsias.com
TEST 31 Solutions

It is also of vital importance to create an ideal micro climate


around the plants.

Greenhouses are framed or inflated structures covered with


transparent or translucent material large enough to grow crops
under partial or fully controlled environmental conditions to get
optimum growth and productivity.

Q Source: Agritech website

71. Which of the following animals are primates?


a) Crocodile and Tortoise
b) Rabbits and Hares
c) Lemurs and Lorises
d) Horses and Zebras

Solution: c)
Learning: Primate is any mammal of the group that includes the
lemurs, lorises, tarsiers, monkeys, apes, and humans.

The order Primates, with its 300 or more species, is the third most
diverse order of mammals, after rodents (Rodentia) and bats
(Chiroptera).

Although there are some notable variations between some primate


groups, they share several anatomic and functional characteristics
reflective of their common ancestry.

When compared with body weight, the primate brain is larger than
that of other terrestrial mammals, and it has a fissure unique to
primates (the Calcarine sulcus) that separates the first and second
visual areas on each side of the brain.

Whereas all other mammals have claws or hooves on their digits,


only primates have flat nails.
www.insightsonindia.com
www.insightsias.com
TEST 31 Solutions

Some primates do have claws, but even among these there is a flat
nail on the big toe (hallux).

In all primates except humans, the hallux diverges from the other
toes and together with them forms a pincer capable of grasping
objects such as branches.

With the exception of humans, who inhabit every continent except


for Antarctica, most primates live in tropical or subtropical regions
of the Americas, Africa and Asia.

Q Source: Past year IES papers

72. The organization "Codex India" is related to


a) Stealth Military operations
b) Food standards in India
c) Developing banking norms
d) Regulation of stock markets

Solution: b)
Learning: The Codex Alimentarius Commission (CAC) was created
FAO and the WHO to essentially develop food standards and guidelines
on related practices.

The main purpose of this Programme is to protect the health of


consumers, ensure fair practices in the food trade, and promote
coordination of all food standards work undertaken by international
governmental and non-governmental organizations.

"Codex India" the National Codex Contact Point (NCCP) for India, is
located at Food Safety and Standards Authority of India (Ministry of
Health and Family Welfare.

It coordinates and promotes Codex activities in India in association with


the National Codex Committee and facilitates India's input to the work of
Codex through an established consultation process.
www.insightsonindia.com
www.insightsias.com
TEST 31 Solutions

Q Source: Agritech website + UPSC past year papers

73. The annual range of surface temperature is much greater in


the North Atlantic and North Pacific oceans than in the southern
oceans. This is due to
a) High cloud formation in the northern oceans
b) Equatorial counter-currents
c) Stratospheric circulation
d) Prevailing winds from land

Solution: d)
Justification: The high ranges in the northern oceans are associated
with the character of the prevailing winds.

In these parts of the oceans, cold winds blow from the continents
toward the oceans and greatly reduce the winter temperatures.

So, annual variations of up to 10C are found sometimes in these


oceans due to low winter temperatures and high summer
temperatures.

In the Atlantic Ocean, the zone of maximum range of annual


temperature of over 8C occurs around 40 N.

From this zone, the range falls of steadily towards north and south.
A minimum annual range of less than 1 C is found at 5 N and a
smaller maximum range is observed in the South Atlantic Ocean at
30 S.

In the North Pacific Ocean, the maximum annual range is higher,


registering a value of about 10 C at 400 N.

In the case of the South Pacific Ocean, the largest annual range of
about 6C is found between latitudes 30 and 40 degree S.
www.insightsonindia.com
www.insightsias.com
TEST 31 Solutions

The greater amount of land in the northern hemisphere is clearly an


important factor in these variations.

Q Source: Improvisation: Past year UPSC papers

74. Biodiversity is not distributed evenly on Earth. It is richest in


a) Arid areas
b) Polar regions
c) Tropics
d) Mediterranean Region

Solution: c)
Justification: Flora and fauna diversity depends on climate, altitude,
soils and the presence of other species.

The tropics are more biologically diverse than other climatic zones is an
accepted fact.

Reasons for this are greater solar insolation, more precipitation,


conditions that are no extreme and conducive to growth of life, lack of
extreme changes as in polar regions (ice ages) etc.

As one approaches Polar Regions one generally finds fewer species.

Q Source: General questions on ecology

75. Consider the following statements.


1. Two important rivers of India originate from here.
2. One of these rivers flows towards north and merges with
another important river flowing towards Bay of Bengal.
3. The other river flows towards Arabian Sea.
www.insightsonindia.com
www.insightsias.com
TEST 31 Solutions

The above refers to which hill?

a) Amarkantak
b) Nallamalla Hills
c) Maikal Hills
d) Rudraprayag Hills

Solution: a)
Justification: River Narmada emerges from Amarkantak Hill of Maikal
Range, and flows westwards towards the Arabian sea.

Son river is largest of southern tributaries of Ganga that originates near


Amarkantak in Madhya Pradesh near the source of Narmada River.
Damodar originates in Chandwa on the Chota Nagpur Plateau.

The Tapi River originates in the Betul district.

Q Source: Past year papers UPSC

76. The Indian Independence League and the Indian National


Army (INA) were formed during
a) Civil Disobedience movement
b) Course of Second World War
c) Non-cooperation movement
d) Swadeshi movement

Solution: b)
Learning: Background: In the Southeast Asia, Japan was making rapid
progress in the War and had already captured Singapore. A large number
of Indian soldiers, were captured as prisoners of war. Also, Japan and
other South Asian countries were refuge for several Indians.

Formation: These Indians under the leadership of Rashbehari Bose


decided to form some kind of organisation to protect their interests
and continue their struggle against the British rule.

The Indiana held a conference at Tokyo in Mardi and decided to


form the Indian Independence league.
www.insightsonindia.com
www.insightsias.com
TEST 31 Solutions

It was decided that the League would raise an Indian national


army. Also, the Indians decided to hold another conference at
Bangkok in June.

It invited Subhash Chandra Bose to lead the movement.

The Indian prisoners of war, captured by the Japanese in


Singapore were placed under an Indian army captain, Mohan
Singh. He formed the first Indian National Army.

Q Source: 12th TamilNadu History Textbook

77.Even if adequate measures are taken to kill the larvae and adult
mosquitoes, which of the following diseases may remain
uninfluenced?
1. Kalazar
2. Filaria
3. Japanese encephalitis
4. Chikungunya

Select the correct answer using the codes below.

a) 2 and 3 only
b) 4 only
c) 1 only
d) 1, 2 and 4 only

Solution: c)
Justification: Chikungunya is a viral disease transmitted to humans by
infected mosquitoes. It causes fever and severe joint pain. So, (b) and (d)
will not be correct.

Japanese encephalitis is a viral disease that infects animals and humans.


It is transmitted by mosquitoes and in humans causes inflammation of
the membranes around the brain. So, (a) also cant be the answer.

Filariasis (or philariasis) is a parasitic disease caused by an infection


with roundworms of the Filarioidea type. These are spread by blood-
www.insightsonindia.com
www.insightsias.com
TEST 31 Solutions

feeding black flies and mosquitoes. So, (a) and (d) also cant be the
answer.

Kala-azar is a chronic and potentially fatal parasitic disease of the viscera


(the internal organs, particularly the liver, spleen, bone marrow and
lymph nodes) due to infection by the parasite called Leishmania
donovani. So, (c) is the only answer.

Q Source: Based on UPSC past year papers

78. Which of the following sites from India was/were recently


added to UNESCO World Heritage list?
1. Chandigarhs Capitol Complex
2. Khangchendzonga National Park
3. Nalanda Mahavihara

Select the correct answer using the codes below.

a) 1 and 2 only
b) 2 and 3 only
c) 1 and 3 only
d) 1, 2 and 3

Solution: d)
Justification: Statement 1: Chandigarhs famed Capitol Complex
designed by Swiss-French architecture Le Corbusier was added in the list
in the Transnational category.

The site was added in the list as part of the UNESCOs recognition
of Le Corbusiers architectural works in different parts of the
world.

The Capitol Complex includes the Punjab and Haryana High Court,
Punjab and Haryana Secretariat and Punjab and Haryana
Assembly.
www.insightsonindia.com
www.insightsias.com
TEST 31 Solutions

Statement 2: KNP was added in the mixed heritage site category of the
list i.e. it exhibits qualities of both natural and cultural significance. With
this, it becomes the first mixed heritage site from India to make it to the
list. KNP covers 25% of Sikkim and is home to a significant number of
endemic, rare and threatened plant and animal species including large
number of bird and mammal species.

Statement 3: We have already covered information about Nalanda


University in past few tests.

Q Source: UNESCO Website

79. Which of the following places on earth is known for largest


annual temperature range?
a) Verkhoyansk in Russia
b) Chennai in TamilNadu
c) Shanghai in China
d) Bharuch in Gujarat

Solution: a)
Justification: The question was very simple if only you knew that
Chennai, Shanghai and Bharuch all are highly influenced by coastal
winds and experience moderate temperature ranges throughout the year.

Moreover, a place in Russia is likely to experience extreme continentality


leading to high variation in temperatures.

The only correct answer can be (a).

The greatest recorded temperature ranges in the world are around the
Siberian `cold pole' in the east of Russia. Temperatures in Verkhoyansk
have ranged 105 deg C from -68 degree Celsius to 37 degree C.

Q Source: Improvisation: CAPF 2016


www.insightsonindia.com
www.insightsias.com
TEST 31 Solutions

80. Consider the following statements.


1. India officially follows and publishes annual reports on Gross
National Happiness (GNH) index.
2. Madhya Pradesh is the first state in India to have a Happiness
Department.

Which of the above is/are correct?

a) 1 only
b) 2 only
c) Both 1 and 2
d) None

Solution: b)
Justification: Statement 1: India follows basic GDP, GNP indicator
approach to economic growth and development, alongwith a stress on
social indicators via measurable targets such as under SHGs.

Statement 2: The department in MP government will work as knowledge


resource centre on the subject of happiness on the lines of Bhutan.

The Happiness Department will work under a working committee


headed by a chairman.

The committee will evaluate the states gross happiness.

The department will prepare guidelines for coordination between


different departments like women and child development, health,
and sports for propagating happiness in the state.

Q Source: http://www.business-standard.com/article/economy-
policy/mp-cabinet-clears-country-s-first-happiness-department-
116071500485_1.html
www.insightsonindia.com
www.insightsias.com
TEST 31 Solutions

81. Consider the following statements.


1. Topslip" is located in the north-east corner of the Manas
National Park.
2. The Orang National Park containing Rhinoceros population is
located on the north bank of the Brahmaputra River.
3. Dibru-Saikhowa National Park is the largest salix swamp forest
in northeast India.
4. Nameri National Park is located in the foothills of the Eastern
Himalayas.

Select the correct answer using the codes below.

a) 1 and 2 only
b) 3 and 4 only
c) 2, 3 and 4 only
d) 1 and 3 only

Solution: c)
Justification: All these national parks have been asked by UPSC in one
form or the other, hence the coverage.

Statement 1: Indira Gandhi Wildlife Sanctuary (formerly called


Anamalai Wildlife Sanctuary), widely known as Top Slip is located at
some distance from Pollachi town. "Topslip" (Annamalai hills) located in
the northeast corner of the park is derived from the local 19th century
practice of sliding timber logs down the hills from here.

The Park and the Sanctuary is under consideration by UNESCO as part


of The Western Ghats World Heritage site.

Statement 2: The Orang National Park is on the north bank of the


Brahmaputra River in the Darrang and Sonitpur districts of Assam. The
park has a rich flora and fauna, including great Indian one-horned
rhinoceros, pigmy hog, elephants, wild buffalo and tigers. It is the only
stronghold of rhinoceros on the north bank of the Brahmaputra

Statement 3: Dibru-Saikhowa National Park is a national wildlife park in


Tinsukia, Assam. It mainly consists of moist mixed semi-evergreen
www.insightsonindia.com
www.insightsias.com
TEST 31 Solutions

forests, moist mixed deciduous forests, canebrakes and grasslands. It is


the largest salix swamp forest (see image below) in northeast India.

Statement 4: Nameri National Park is in the foothills of the Eastern


Himalayas in the Sonitpur District of Assam. It hosts large elephant
population. It is also an ideal habitat for a host of other animals
including the tiger, leopard, sambar, dhole (the Asiatic wild dog), pygmy
hog, muntjac, gaur, wild boar, sloth bear, Himalayan black bear, capped
langur and Indian giant squirrel.

Q Source: National parks

82. Which of the following regarding the procedure of motion of


no-confidence in Council of Ministers is INCORRECT?
a) It needs to state the reasons for its adoption in the Lok
Sabha.
b) The motion will not be admitted if less than 50 MPs of Lok
Sabha support it.
c) If the Speaker grants leave to the motion, a decision on it
need not be taken immediately.
d) None of the above

Solution: a)
www.insightsonindia.com
www.insightsias.com
TEST 31 Solutions

Justification: A motion expressing want of confidence in the Council of


Ministers may be made subject to the following restrictions, namely:-

Leave to make the motion shall be asked for by the member when
called by the Speaker;

The member asking for leave shall, give to the Secretary-General a


written notice of the motion which he proposes to move.

The motion does not need to state the reasons for its adoption in
the Lok Sabha. So, (a) is incorrect.

If the Speaker is of opinion that the motion is in order, he shall


read the motion to the House and shall request those members
who are in favour of leave being granted to rise in their places.

If not less than fifty members rise accordingly, the Speaker shall
declare that leave is granted and that the motion will be taken up
on such day, not being more than ten days from the date on which
the leave is asked for as he may appoint.

If less than fifty members rise, the Speaker shall inform the
member that he has not the leave of the House.

Q Source: Indian Polity


http://www.archive.india.gov.in/knowindia/culture_heritage.php?id=33

83. Which of the following is/are the applications of Geotextiles?


1. It can improve soil strength and help in erosion control.
2. It has good tensile strength which allows it to be used in water
structures.
3. It is highly biodegradable and thus used to increase humus
content of the soil.

Select the correct answer using the codes below.

a) 1 and 2 only
b) 2 and 3 only
c) 1 and 3 only
d) 1, 2 and 3
www.insightsonindia.com
www.insightsias.com
TEST 31 Solutions

Solution: a)
Justification: It is a permeable geosynthetic comprised solely of
textiles. Geotextiles are used with foundation, soil, rock, earth, or any
other geotechnical engineering-related material as an integral part of
human-made project, structure, or system.

Statement 1 and 2: These are used in reinforcement of embankments or


in constructional work.

The fabrics in geo textiles are permeable fabrics and are used with
soils having ability to separate, filter, protect or drain.

The application areas include civil engineering, earth and road


construction, dam engineering, soil sealing and in drainage
systems. The fabric used in it must have good strength, durability,
low moisture absorption and thickness.

Statement 3: They are indeed textiles in the traditional sense, but


consist of synthetic fibers rather than natural ones such as cotton, wool,
or silk. Thus biodegradation is not a problem.

These synthetic fibers are made into a flexible, porous fabric by


standard weaving machinery or are matted together in a random,
or nonwoven, manner. Some are also knit.

They are porous to water flow across their manufactured plane and
also within their plane, but to a widely varying degree.

For more information, see the THE BASIC FUNCTION OF


GEOTEXTILE under
http://www.engr.utk.edu/mse/Textiles/Geotextiles.htm

Q Source: Ministry of Textiles website Technical textiles

84. The purpose of genetically modifying Bt-cotton by inserting


Bacillus thuringiensis bacterium was to make cotton
www.insightsonindia.com
www.insightsias.com
TEST 31 Solutions

a) Drought-resistant
b) High-yielding
c) Pest-resistant
d) Long-duration crops

Solution: c)
Learning: Bt cotton is a genetically modified organism (GMO) cotton
variety, which produces an insecticide to bollworm.

Bacillus thuringiensis (Bt) is a soil bacterium that produces insecticidal


toxins. Genes from Bt can be inserted into crop plants to make them
capable of producing an insecticidal toxin and therefore resistant to
certain pests.

It was inserted into Bt-cotton as well as Bt-Brinjal.

Q Source: NCERT 12th Biology

85. Privy purse given to the princely states in post-independent


India was
a) An award of recognition to those royal households who laid
their lives in the freedom struggle
b) A grant given by the Government of India to the erstwhile
Princes of India, which was later abolished
c) Protection given by the princely states to the new formed
government at the Centre
d) A group of erstwhile provinces which contributed to the
Central treasury

Solution: b)
Learning: Privy Purse in India was a payment that was made to the
royal families of the former princely states of India.

The Privy Purse was created as part of the agreements made by


them to merge with Union of India in the year 1947.
www.insightsonindia.com
www.insightsias.com
TEST 31 Solutions

Later in 1949 the various Indian Princes merged their princely


states whereby the individual independent rulers lost their ruling
rights.
Until the Constitutional Amendment of 1971 was passed the Privy
Purse in India was sustained to the royal families.
Through this amendment, which was implemented after an
extensive legal conflict, the privileges and allowances, provided by
the Central Government of India to the Indian princes, ceased to
exist.

Q Source: NCERT 12th History + Past year papers UPSC

86. The Ozone Cell housed in the Ministry of Environment,


Forest and Climate Change (MoEF&CC)
a) Promotes awareness about ozone depletion
b) Engages in the implementation of the Montreal protocol
c) Researches as a scientific body on ozone and its depletion in
the atmosphere
d) None of the above

Solution: b)
Learning: Government of India has entrusted the work relating to the
ozone layer protection and implementation of the Montreal Protocol on
Substances the Ozone Layer to the Ministry of Environment, Forest and
Climate Change (MoEF&CC).

The Ministry has set up an Ozone Cell as a National Ozone Unit (NOU)
to render necessary services for effective and timely implementation of
the Montreal Protocol and its ODS phase-out program in India.

Currently, the Ozone Cell is engaged in phase-out of production and


consumption of next category of chemicals, Hydrochlorofluorocarbons
(HCFCs) with an accelerated phase-out schedule as per the Montreal
Protocol.
www.insightsonindia.com
www.insightsias.com
TEST 31 Solutions

Q Source: MOEF&CC Website + Past year UPSC papers

87. Industry generating the maximum employment of labour in


India is
a) Steel industry
b) Fertilizers industry
c) Cotton and Textile industry
d) Cement industry

Solution: c)
Learning: The textile industry in India traditionally, after agriculture,
is the only industry that has generated huge employment for both skilled
and unskilled labor in textiles.

The textile industry continues to be the second largest employment


generating sector in India. It offers direct employment to over 35
million in the country.

In 2013-14, most of the jobs, 88% of the total, were generated by


textiles and IT-BPO sectors.

In fact, 62% of total jobs created in the textiles sector was


contributed by the exporting units.

Also, the Small Sector Industries (SSI) Sector in India creates


largest employment opportunities for the Indian populace, next
only to Agriculture.

Q Source: General economic statistics


www.insightsonindia.com
www.insightsias.com
TEST 31 Solutions

88. Reserve deposit ratio (rdr) is the proportion of the total


deposits that is kept by the commercial banks as reserves. Why
keeping reserves is costly for banks?
1. RBI charges them a fee on the total amount of reserve deposits.
2. They could lend these reserves for return earning investments.

Which of the above is/are correct?

a) 1 only
b) 2 only
c) Both 1 and 2
d) None

Solution: b)
Justification: Statement 1: No fee is charged by RBI, neither RBI gives
a return on some of the vault cash kept by banks with RBI, for e.g. CRR
earns no interest.

RBI requires commercial banks to keep reserves in order to ensure that


banks have a safe cushion of assets to draw on when account holders
want to be paid.

Statement 2: If there was reserve requirement, this money could have


been invested elsewhere by the banks.

For e.g. the Cash Reserve Ratio kept with RBI can yield handsome
returns if invested in the security markets.

Q Source: 12th NCERT Macroeconomics

89. Declaration of establishing National Parks can be made by


a) Both Central and State governments
b) Central government only
c) State and local governments only
d) Local governments only
www.insightsonindia.com
www.insightsias.com
TEST 31 Solutions

Solution: a)
Learning: As per the Wildlife Act, Whenever the State Government
finds an area ecologically important it can be constituted as a national
park.

The Act also gives this power to Central governments to declare both
wildlife sanctuaries and national parks.

But, no alteration of the boundaries of a National Park shall be made


except on a resolution passed by the Legislature of the State.

The act also has provisions for protection of the reserves, improvement
and better management of wildlife, management of activities etc.

Q Source: http://www.moef.nic.in/legis/wildlife/wildlife1c4.html

90. Consider the following matches of Centre of Handicrafts with


the state they are found in.
1. Mon : Arunachal Pradesh
2. Nalbari : Assam
3. Pasighat : Meghalaya
4. Tura : Nagaland

Select the correct answer using the codes below.

a) 1 and 4 only
b) 2 only
c) 2 and 3 only
d) 1, 3 and 4 only

Solution: b)
Justification: Statement 1: Mon is a town and a town area committee
in Mon district in Nagaland. This district is also known for Headhunting
as it was practised in historic times.

Statement 2: Nalbari is a small town in Nalbari district of Assam.

Statement 3: Pasighat is the headquarters of East Siang district in


Arunachal Pradesh. Pasighat is Arunachal's oldest town.
www.insightsonindia.com
www.insightsias.com
TEST 31 Solutions

Pasighat has been selected as one of the hundred Indian cities to be


developed as a smart city under PM Narendra Modi's flagship Smart
Cities Mission.

Statement 4: Tura is a hilly town and a municipality in West Garo Hills


district in Meghalaya. Tura is a cultural and administrative centre of the
Garo tribes.

Q Source: Based on past year UPSC papers

91.Juba in South Sudan was recently in news because of


a) Discovery of large quantity of gas hydrates in its limestone
shells
b) Evacuation of Indian citizens due to ongoing crisis in the
region
c) Economic boom like conditions in Sudan and neighbouring
nations
d) Prime Ministers visit announcing a major investment
package for the region

Solution: b)
Learning: The South Sudanese Civil War is a conflict in South Sudan
between forces of the government and opposition forces.

The fighting, between those who supported President Salva Kiir, a


member of the Dinka tribe, and recently ousted Vice President
Riek Machar, a Nuer, has sparked a major humanitarian crisis.

The violence has rapidly escalated since it began and has spread to
other parts of the country, leaving more than 1.35 million people
displaced and a staggering 4.9 million people in need of
humanitarian assistance (OCHA).

A peace agreement known as the "Compromise Peace Agreement"


was signed in Ethiopia under threat of United Nations sanctions
for both sides in August 2015.
www.insightsonindia.com
www.insightsias.com
TEST 31 Solutions

Violence erupted again in July 2016 after an attack outside of


where President Salva Kiir and Riek Machar, the Vice President
and leader of SPLM-IO, were meeting in Juba.

President Salva Kiir and first Vice-President Riek Machar ordered


a ceasefire after days of intense violence.

Indian Air Force evacuated Indian citizens from the country under
Operation Sankat Mochan.

Q Source: http://www.nytimes.com/2016/07/23/world/africa/south-
sudan-kiir-machar-peace-civil-war.html

92. The Distribution function is an important part of public


finance. This function is carried out by the Government using?
a) Taxation and Public expenditure
b) Distributing only freebies to poor and underprivileged
c) Tight control of the Central Bank
d) Easing caste mobility

Solution: a)
Learning: In a mixed economy, apart from the private sector, there is
the government which plays a very important role.

Through its tax and expenditure policy, the government attempts to


bring about a distribution of income that is considered fair by society.

The government affects the personal disposable income of households by


making transfer payments and collecting taxes and, therefore, can alter
the income distribution. This is the distribution function.

Q Source: 12th NCERT: Macroeconomics


www.insightsonindia.com
www.insightsias.com
TEST 31 Solutions

93. The term Prisoners Dilemma is associated with


a) Pharmaceutical production and patent rights
b) Intelligence wings of the Government operating overseas
c) A case under the Game Theory
d) Hostage situation in aircraft hijacks only

Solution: c)
Learning: The prisoners dilemma is the best-known game of strategy
in social science. It helps us understand what governs the balance
between cooperation and competition in business, in politics, and in
social settings.

In the traditional version of the game, the police have arrested two
suspects and are interrogating them in separate rooms. Each can
either confess, thereby implicating the other, or keep silent.

No matter what the other suspect does, each can improve his own
position by confessing. If the other confesses, then one had better
do the same to avoid the especially harsh sentence that awaits a
recalcitrant holdout.

If the other keeps silent, then one can obtain the favorable
treatment accorded a states witness by confessing. Thus,
confession is the dominant strategy (see game theory) for each. But
when both confess, the outcome is worse for both than when both
keep silent.

The prisoners dilemma has applications to economics and business, for


e.g. two firms deciding on their pricing strategy, or two nations engaged
in arms race.

Q Source: John Nash Game theory asked many times in UPSC


www.insightsonindia.com
www.insightsias.com
TEST 31 Solutions

94. Chir, pine, deodar, spruce and silver fir are most likely to
occur in
a) Tropical Rainforests of Kerala
b) Shrub forests of Western India
c) Deciduous forests of Central India
d) Temperate vegetation of Western Himalayas

Solution: d)
Learning: The Western Himalayan region extends from Kashmir to
Kumaon. Its temperate zone is rich in forests of chir, pine, other conifers
and broad-leaved temperate trees.

Higher up, forests of deodar, blue pine, spruce and silver fir occur.

The alpine zone extends from the upper limit of the temperate zone
of about 4,750 metres or even higher.

The characteristic trees of this zone are high-level silver fir, silver
birch and junipers.

The eastern Himalayan region extends from Sikkim eastwards and


embraces Darjeeling, Kurseong and the adjacent tract.

The temperate zone has forests of oaks, laurels, maples,


rhododendrons, alder and birch. Many conifers, junipers and
dwarf willows also occur here.

Q Source: NCERT 8th Geography

95. Consider the following statements.


1. Assam shares a border with Bhutan and Bangladesh.
2. West Bengal shares a border with Bhutan and Nepal.
3. Mizoram shares a border with Bangladesh and Myanmar.
4. Nagaland shares border with China.

Select the correct answer using the codes below.


www.insightsonindia.com
www.insightsias.com
TEST 31 Solutions

a) 1 and 2 only
b) 1 and 4 only
c) 1, 2 and 3 only
d) 2 and 4 only

Solution: c)
Justification: The map below. You should go through the map of
North-east India and South-east Asia even more carefully. A lot of
questions are being asked from this area in UPSC.

Q Source: Map based questions

96. Consider the following statements.


1. The Advocate General of a State in India is appointed by the
President of India on the recommendation of the Governor of
the concerned State.
2. High Courts have not been granted any power under the
constitution and derive all their authority from Civil and
Criminal Procedure Codes.
www.insightsonindia.com
www.insightsias.com
TEST 31 Solutions

Which of the above is/are correct?

a) 1 only
b) 2 only
c) Both 1 and 2
d) None

Solution: d)
Justification: Statement 1: The Office of the Advocate General is a
constitutional office created under Article 165 of the Constitution of
India. The Governor of the State appoints a person who is qualified to be
appointed as a judge of the High Court as Advocate General of the State.

Statement 2: The High court derive their writ jurisdiction, judicial


review powers, original jurisdiction etc from the constitution.

Q Source: Indian Polity: M Laxmikanth

97. Consider the following statements.

Assertion (A): A longer column of flute produces lower notes and a


shorter column produces higher notes.

Reason (R): The length of a flute column determines the frequency of the
sound waves produced by it.

In the context of the above, which of these is correct?

a) A is correct, and R is an appropriate explanation of A.


b) A is correct, but R is not an appropriate explanation of A.
c) A is correct, but R is incorrect.
d) Both A and R are incorrect.

Solution: a)
Justification: Sound is made up of vibrations or waves. These waves
have a speed or frequency that they vibrate at. The pitch of the note
changes depending on the frequency of these vibrations. The higher the
frequency of the wave, the higher the pitch of the note will sound.
www.insightsonindia.com
www.insightsias.com
TEST 31 Solutions

In a flute, as the pressure waves move back and forth inside the
column of the flute, they disturb the air at both open ends of the
flute.

The distance between both open ends determines the length of the
air column. Flutists change the length of the air column by opening
or closing keys. The first open end of the flute, the embouchure
hole, is always in the same position, but second open end of the
flute is determined by the last closed hole.

So, the length of the column determines the frequency of the


waves. A longer column produces waves with lower frequencies. A
shorter column produces waves with higher frequencies. In other
words, a longer column produces lower notes and a shorter column
produces higher notes.

Q Source: Improvisation: UPSC past year papers

98. The Nehru Report favoured


1. Dominion Status for India
2. Partition of India
3. A completely Centralized government in India

Select the correct answer using the codes below.

a) 1 and 2 only
b) 1 only
c) 2 and 3 only
d) 1 and 3 only

Solution: b)
Justification: A meeting of various Indian parties, called as an "All
Parties Conference" in 1928 appointed a Committee to consider and
determine the principles of the Constitution for India. It was headed by
Motilal Nehru.

The Report favoured:


www.insightsonindia.com
www.insightsias.com
TEST 31 Solutions

Dominion Status as the next immediate step

Full responsible government at the centre.

Autonomy to the provinces.

Clear cut division of power between the centre and the provinces.

A bicameral legislature at the centre.

There will be no separate electorate.

The draft report also defined the citizenship and fundamental


rights.

However, the leader of the Muslim League, Mohammad Ali Jinnah


regarded it as detrimental to the interests of the Muslims.

Jinnah convened an All India Conference of the Muslims where he drew


up a list of Fourteen Points as Muslim League demand.

Q Source: 12th TamilNadu History Textbook

99. Which of the following was NOT a provision under the


Government of India Act of 1858?
a) Indian administration came under the direct control of the
Crown.
b) The Secretary of State was to be a member of the British
cabinet.
c) The Governor General of India was also made the Viceroy of
India.
d) The government accepted the right of princely states as
independent political units.

Solution: d)
Learning: Following are the main provisions of the Act:

East India Companys rule came to an end and the Indian


administration came under the direct control of the Crown.
www.insightsonindia.com
www.insightsias.com
TEST 31 Solutions

In England, the Court of Directors and Board of Control were


abolished. In their place came the Secretary of State for India and
India Council were established.

The Secretary of State would be a member of the British cabinet.


Sir Charles Wood was made the first Secretary of State for India.
India Council consisting of 15 members would assist him.

The Governor General of India was also made the Viceroy of India.
The first Viceroy of India was Lord Canning.

All the previous treaties were accepted and honoured by the Act

Q Source: 12th TamilNadu History Textbook

100. A value of 1 for Gini Coefficient in a country implies that


a) There is perfectly income equality in the population.
b) One person owns all the income in the country.
c) Income is equally distributed between the rich and the poor.
d) There is no poverty in the economy.

Solution: b)
Justification & Learning: The Gini coefficient is often used to
measure income inequality. Here, 0 corresponds to perfect income
equality (i.e. everyone has the same income) and 1 corresponds to perfect
income inequality (i.e. one person has all the income, while everyone else
has zero income).

The Gini coefficient can also be used to measure wealth inequality. This
use requires that no one has a negative net wealth.
www.insightsonindia.com
www.insightsias.com
TEST 31 Solutions

The Gini coefficient is defined as a ratio of the areas on the Lorenz curve
diagram. If the area between the line of perfect equality and Lorenz
curve is A, and the area under the Lorenz curve is B, then the Gini
coefficient is A/(A+B).

Q Source: http://data.worldbank.org/indicator/SI.POV.GINI

You might also like